Nothing Special   »   [go: up one dir, main page]

PW Only IAS Prelims 2024 Test 14 With Solution

Download as pdf or txt
Download as pdf or txt
You are on page 1of 54

https://upscpdf.com/ https://upscpdf.

com/

Prelims Wallah
Test
Test-14 of 38
(Economy Part II)

TARGET 2024

SCAN HERE FOR TEST SERIES ACCESS

https://upscpdf.com/ https://upscpdf.com/
https://upscpdf.com/ https://upscpdf.com/

Prelims Wallah Test 2024


Test 14: Economy Part II
1. Who among the following first set up a 3. Allotting special drawing rights (SDRs)
National Planning Committee to focus on the to the member nations
development of industries in India? How many of the above are the main
(a) Mahatma Gandhi functions of the International Monetary Fund
(b) Jawaharlal Nehru (IMF)?
(c) Vallabhbhai Patel (a) Only one
(d) Subhas Chandra Bose (b) Only two
(c) All three
2. Consider the following statements regarding (d) None
currency convertibility in India:
1. India has full convertibility in its current 5. Consider the following conventions:
account. 1. Minamata Convention on Mercury.
2. Individuals in India are allowed to invest 2. Stockholm Convention on Persistent
an unlimited amount in foreign assets, Organic Pollutants
shares, etc. 3. United Nations Convention on
3. The S.S. Tarapore Committee's Biological Diversity
recommendations in 1997 led to full 4. United Nations Convention to Combat
capital account convertibility in India. Desertification
Which of the statements given above are The Global Environmental Facility (GEF)
incorrect? serves as a financial mechanism for how
(a) 1 and 2 only many of the above conventions?
(b) 2 and 3 only (a) Only one
(c) 1 and 3 only (b) Only two
(d) 1, 2 and 3 (c) Only three
(d) All four
3. Consider the following:
1. Certification of insurance companies 6. Consider the following statements:
2. Protecting the interest of the 1. Insurance penetration is the ratio of
policyholders premium underwritten in a given year to
3. Adjudication of disputes the total population.
How many of the above are the functions of 2. Insurance density is the ratio of
the Insurance Regulatory and Development premiums underwritten in a given year to
Authority? the gross domestic product (GDP).
(a) Only one Which of the statements given above are
(b) Only two correct?
(c) All three (a) 1 only
(d) None (b) 2 only
4. Consider the following: (c) Both 1 and 2
1. Exchange rate regulation (d) Neither 1 nor 2
2. Purchasing short-term foreign currency
liabilities of the member nations

https://upscpdf.com/ https://upscpdf.com/
https://upscpdf.com/ https://upscpdf.com/

7. Which one of the following reports is (a) Only one


released by the International Monetary Fund (b) Only two
(IMF)? (c) Only three
(a) Global Financial Stability Report (d) All four
(b) World Development Report
(c) International Debt Statistics 12. With reference to Foreign Direct Investment
(d) Ease of Doing Business Report in the Insurance sector, consider the
following statements:
8. Consider the following pairs: 1. The insurance sector was opened up for
Economic Plans Formulated by the private sector in 1982.
1. Sarvodaya Plan Jaiprakash Narayan 2. Currently foreign investment ceiling in
2. Gandhian Plan M.N.Roy the insurance sector is set at 49%.
3. People’s Plan M.G. Ranade Which of the statements given above are
4. Bombay Plan Capitalists correct?
How many of the above pairs are correctly (a) 1 only
matched? (b) 2 only
(a) Only one (c) Both 1 and 2
(b) Only two (d) Neither 1 nor 2
(c) Only three
(d) All four 13. Which one of the following is the objective
of the “Antardrishti” initiative by the Reserve
9. ‘Global Liveability Index’ is released by Bank of India?
which one of the following organisations? (a) Financial Inclusion
(a) World Bank (b) Inflation Control
(b) World Economic Forum (c) Credit rationing
(c) Economic Intelligence Unit (d) Prompt Corrective Action framework
(d) Ministry of Housing and Urban Affairs
14. Consider the following institutions:
10. Extended Fund Facility (EFF) and Financial 1. The Multilateral Investment Guarantee
Transactions Plan (FTP) is often used in the Agency (MIGA)
context of which one of the following? 2. The International Finance Corporation
(a) Free Trade Agreement (FTA) (IFC)
(b) World Trade Organisation 3. The International Development Agency
(c) World Bank (IDA)
(d) International Monetary Fund 4. The International Centre for Settlement
of Investment Disputes (ICSID)
11. Consider the following countries: How many of the above institutions have
1. Saudi Arabia India as a member?
2. Syria (a) Only one
3. Lebanon (b) Only two
4. Jordan (c) Only three
How many of the above countries border (d) All four
Israel?

https://upscpdf.com/ https://upscpdf.com/
https://upscpdf.com/ https://upscpdf.com/

15. Which of the following statements is correct 18. Consider the following statements regarding
with reference to the Monkeypox disease? the Asian Infrastructure Investment Bank
1. It is a viral illness caused by a species of (AIIB):
the genus Orthopoxvirus. 1. It offers only the sovereign (government)
2. It can be transmitted to humans through financing and not the private financing
physical contact with an infected person. for sound and sustainable projects.
3. Vaccine is not available for its prevention 2. It does not have any members from
Select the correct answer using the code outside Asia.
given below: Which of the statements given above are
(a) 1 and 2 only correct?
(b) 2 and 3 only (a) 1 only
(c) 1 and 3 only (b) 2 only
(d) 1, 2 and 3 (c) Both 1 and 2
(d) Neither 1 nor 2
16. Consider the following statements:
19. Consider the following statements with
1. The first centralised and integrated Five-
reference to the National Consumer Disputes
Year plan was implemented by the Soviet
Redressal Commission (NCDRC):
Union.
1. It is a quasi-judicial commission in India
2. The economic and social planning is a
that was set up in 1988 under the
State subject in the Indian Constitution.
Consumer Protection Act, of 1986.
Which of the statements given above are
2. There lies no appeal to any order of the
correct?
commission.
(a) 1 only
Which of the statements given above are
(b) 2 only
correct?
(c) Both 1 and 2
(a) 1 only
(d) Neither 1 nor 2
(b) 2 only
(c) Both 1 and 2
17. Consider the following events:
(d) Neither 1 nor 2
1. Nationalisation of life insurance business
2. Establishment of General Insurance 20. With reference to the Asian Development
Corporation of India (GIC) Bank (ADB), consider the following
3. Establishment of the Insurance statements:
Regulatory and Development Authority 1. ADB was established in 2002 and has
(IRDA) headquarters in Tokyo, Japan.
Which one of the following represents the 2. India is the largest shareholder in ADB.
correct chronological order of the above 3. Members of ADB cannot join the Asian
events from past to present? Infrastructure Investment Bank (AIIB).
(a) 1-3-2 Which of the statements given above are
(b) 2-3-1 incorrect?
(c) 1-2-3 (a) 1 only
(d) 3-2-1 (b) 2 and 3 only
(c) 1 and 3 only
(d) 1, 2 and 3

https://upscpdf.com/ https://upscpdf.com/
https://upscpdf.com/ https://upscpdf.com/

21. The term ‘Brumbrella’ was recently seen in 2. The reserve tranches that countries hold
the news in the context of which one of the with the IMF are considered their
following? facilities of last resort.
(a) Javelin throw Which of the statements given above are
(b) Badminton correct?
(c) Football (a) 1 only
(d) Cricket (b) 2 only
(c) Both 1 and 2
22. Consider the following statements: (d) Neither 1 nor 2
1. The First Five-Year Plan of India was
launched in 1951 and was based on the 25. Which of the following statements is correct
Harrod-Domar model. with reference to the steel industry in India?
2. The main focus of the First Five-Year 1. India is among the largest producers of
Plan was the development of the steel in the world.
secondary sector. 2. India is home to the seventh largest
Which of the statements given above are reserves of iron ore in the world.
correct? 3. Rajasthan is the largest producer of steel
(a) 1 only in the country.
(b) 2 only Select the correct answer using the code
(c) Both 1 and 2 given below:
(d) Neither 1 nor 2 (a) 1 and 2 only
(b) 2 and 3 only
23. Consider the following statements with (c) 1 and 3 only
reference to Sovereign Gold Bonds: (d) 1, 2, and 3
1. They are government securities
denominated in grams of gold. 26. Consider the following statements with
2. A minor cannot invest in these bonds. reference to the Stockholm International
3. An individual investor can buy only 4 kg Peace Research Institute (SIPRI):
worth of gold every year. 1. It is an independent international institute
How many of the statements given above are dedicated to research into conflict,
correct? armaments, arms control, and
(a) Only one disarmament.
(b) Only two 2. SIPRI was established by the United
(c) All three Nations Economic and Social Council
(d) None (ECOSOC).
Which of the statements given above are
24. With reference to the reserve tranche, correct?
consider the following statements: (a) 1 only
1. The reserve tranche is an emergency (b) 2 only
account that IMF members can access (c) Both 1 and 2
without agreeing to conditions or paying (d) Neither 1 nor 2
a service fee.

https://upscpdf.com/ https://upscpdf.com/
https://t.me/visioniastestseries2024
https://upscpdf.com/ https://upscpdf.com/

27. Contingent Reserve Arrangement (CRA) is (d) It is mandatory for every corporate house
related to additional liquidity protection for to evaluate its readiness for making the
member-nations of which of the following? workplace accessible.
(a) Asian Development Bank
(b) International Monetary Fund 30. The awardee for the Gandhi Peace Prize is
(c) New Development Bank selected by a five-member jury consisting of
(d) World Bank which of the following?
1. Prime Minister
28. With reference to the ‘National Development 2. Chief Justice of India
Council (NDC)’, consider the following 3. Leader of Opposition
statements: 4. Union Minister of Culture
1. The National Development Council is a Select the correct answer using the code
constitutional body established to given below:
facilitate cooperative federalism in (a) 1 and 2 only
planning. (b) 1, 2 and 3 only
2. It is presided over by the President of (c) 1 and 3 only
India, and its members include the Chief (d) 1, 2, 3 and 4
Ministers of all states.
3. It has the power to directly allocate funds 31. Consider the following statements with
to states from the Consolidated Fund of reference to the Central Electricity Authority
India for the execution of centrally- (CEA):
sponsored schemes. 1. It is a Statutory Body constituted under
How many of the statements given above are the Electricity (Supply) Act, 1948.
correct? 2. It advises the Central Government on
(a) Only one matters relating to the national electricity
(b) Only two policy.
(c) All three Which of the statements given above are
(d) None correct?
(a) 1 only
29. Which one of the following statements is (b) 2 only
incorrect about the ‘Inclusiveness and (c) Both 1 and 2
Accessibility Index’? (d) Neither 1 nor 2
(a) The Index is launched as a part of the
Sugamya Bharat Abhiyan. 32. Consider the following statements regarding
(b) The Index is prepared in collaboration Insurance In India:
with The Federation of Indian Chambers 1. Properties in India cannot be insured with
of Commerce and Industry (FICCI). a foreign insurer.
(c) The Index has been prepared with a 2. The orders given by the Insurance
holistic approach and may be applied and Regulatory and Development Authority
used by organisations irrespective of (IRDA) are final, and there lies no appeal
their size and scale. against these orders.

https://upscpdf.com/ https://upscpdf.com/
https://t.me/visioniastestseries2024
https://upscpdf.com/ https://upscpdf.com/

Which of the statements given above are 2. It plays a catalytic role in mobilising
correct? government investment in developed
(a) 1 only countries only.
(b) 2 only Which of the statements given above are
(c) Both 1 and 2 incorrect?
(d) Neither 1 nor 2 (a) 1 only
(b) 2 only
33. Consider the following statements regarding (c) Both 1 and 2
BRICS Bank: (d) Neither 1 nor 2
1. It was created by the Fortaleza
Declaration. 36. With reference to the ‘Second Five-year
2. It supports projects in both public and plan’, consider the following statements:
private sectors through loans and other 1. P.C. Mahalanobis played an important
instruments. role in its formulation.
Which of the statements given above are 2. The plan emphasised rapid
correct? industrialisation, particularly in heavy
(a) 1 only and basic industries.
(b) 2 only 3. It put more emphasis on capital goods,
(c) Both 1 and 2 which resulted in a scarcity of essential
(d) Neither 1 nor 2 commodities.
Which of the statements given above are
34. Which of the following factors are correct?
responsible for the declining Female Labour (a) 1 and 2 only
Force Participation Rate in the economy? (b) 3 only
1. Delayed entry of rural women into the (c) 2 and 3 only
Labour market due to pursuing higher (d) 1, 2 and 3
education.
2. Absence of quality jobs. 37. Consider the following:
3. Relatively higher responsibilities of 1. Deposits of foreign Governments
unpaid work. 2. Deposits of Central/State Governments
Select the correct answer using the code 3. Inter-bank deposits
given below. How many of the above are insured by
(a) 1 and 2 only Deposit Insurance and Credit Guarantee
(b) 2 and 3 only Corporation?
(c) 1 and 3 only (a) Only one
(d) 1, 2 and 3 (b) Only two
(c) All three
35. With reference to the International Finance (d) None
Corporation (IFC), consider the following
statements:
1. It is the private arm of the World Bank.

https://upscpdf.com/ https://upscpdf.com/
https://t.me/visioniastestseries2024
https://upscpdf.com/ https://upscpdf.com/

38. The ‘Global Economic Prospects’ report is 41. Consider the following statements regarding
released by which one of the following the state of agriculture in India:
organisations? 1. The share of the agriculture sector in
(a) Organisation for Economic Co-operation India's GDP has seen a consistent rise
and Development since independence.
(b) World Economic Forum 2. Around 55% of India's population still
(c) International Monetary Fund depends on the agriculture sector for
(d) World Bank livelihood.
3. Agriculture carries no burden of
39. Consider the following statements regarding individual income tax.
the agricultural cropping seasons in India: 4. The productivity gap between on-the-
1. The Indian cropping season is classified field and ideal farm practices has
into two main seasons, i.e. kharif and significantly increased in recent years.
rabi. Which of the statements given above are
2. The term Zaid refers to crops grown correct?
between July and October. (a) 1 and 2 only
3. Kharif crops include wheat, barley, and (b) 2 and 3 only
oats. (c) 1, 2 and 3 only
How many of the statements given above are (d) 2 and 4 only
correct? 42. Which of the following is the aim of the
(a) Only one National Land Records Modernization
(b) Only two Programme (NLRMP) initiated in 2008?
(c) All three (a) Facilitating large-scale land acquisition
(d) None for industrial projects
(b) Digitisation of land records
40. Which among the following are the efforts (c) Prohibiting land leasing norms to
undertaken by the Government to help safeguard farmers' interests
formalize the economy? (d) Promoting corporate farming for food
1. Introduction of GST security
2. Direct Benefit transfer of 43. Consider the following statements:
subsidies/scholarships 1. Family planning was a major target of
3. Extending social security coverage this Development Plan.
4. Digitisation of payments 2. The Plan was based on the Gadgil
Select the correct answer using the code strategy.
given below. 3. Emphasis was on the growth rate of
(a) 1 and 2 only agriculture to enable other sectors to
(b) 1, 2 and 4 only move forward.
(c) 3 and 4 only Which of the following Five-Year plans is
(d) 1, 2, 3 and 4 best described by the above statements?
(a) Third Five-Year plan
(b) Fourth Five-Year plan
(c) Second Five-Year plan
(d) Ninth Five-Year plan

https://upscpdf.com/ https://upscpdf.com/
https://t.me/visioniastestseries2024
https://upscpdf.com/ https://upscpdf.com/

44. Consider the following statements with 46. Which of the following terms is used to
reference to Green Climate Fund: describe the maximum size of an agricultural
1. It was established by UN Framework holding that must be possessed and owned by
Convention on Climate Change member a family?
countries in 2010. (a) Economic Holding
2. The Fund allocates its resources in low- (b) Family Holding
emission and climate-resilient (c) Optimum Holding
development projects in developing (d) Operational Holding
countries.
3. Stockholm convention on Persistent 47. Consider the following statements with
Organic Pollutants is specifically funded reference to the National Action Plan on
by this fund. Climate Change:
How many of the statements given above are 1. There are twelve ‘National Missions’
correct? which form the core of the plan.
(a) Only one 2. An objective of the plan is to achieve
(b) Only two national growth and poverty alleviation
(c) All three while ensuring ecological sustainability.
(d) None Which of the statements given above are
correct?
45. Consider the following: (a) 1 only
1. To reduce the emissions intensity of its (b) 2 only
GDP by 33 to 35 percent by 2030 from (c) Both 1 and 2
2005 level. (d) Neither 1 nor 2
2. To create an additional carbon sink of 2.5
to 3 billion tonnes of CO2 equivalent 48. Consider the following statements about the
through additional forest and tree cover Samagra Shiksha scheme:
by 2030. 1. It was launched by subsuming two
3. To achieve about 40 per cent cumulative schemes, namely Sarva Shiksha Abhiyan
electric power installed capacity from (SSA) and Rashtriya Ucchatar Shiksha
non-fossil fuel based energy resources. Abhiyan(RUSA).
How many of the above were included in 2. The scheme includes an annual school
India’s Intended Nationally Determined grant to look after a variety of needs of
Contribution (INDCs) submitted at COP 21 the school.
of United Nations Framework Convention on Which of the statements given above are
Climate Change? correct?
(a) Only one (a) 1 only
(b) Only two (b) 2 only
(c) All three (c) Both 1 and 2
(d) None (d) Neither 1 nor 2

https://upscpdf.com/ https://upscpdf.com/
https://t.me/visioniastestseries2024
https://upscpdf.com/ https://upscpdf.com/

49. The concept of micro insurance has been 53. ‘The Atlantic Declaration’ is often mentioned
developed by which one of the following in the news with reference to which of the
private companies in India? following countries?
(a) HDFC Standard Life Insurance (a) United Kingdom and Canada
(b) Aviva Life Insurance (b) France and United Nations
(c) Canara Bank Insurance (c) United States and United Kingdom
(d) General Insurance Corporation of India (d) Canada and France
(GIC)
54. With reference to the United Nations
50. During which Five-Year plan did economic Conference on Sustainable Development,
reforms, including liberalisation and consider the following:
globalisation policies, gain prominence in 1. It took place in Rio de Janeiro, Brazil in
India? 2012.
(a) Fourth Five-Year Plan 2. Governments adopted the 10-year
(b) Seventh Five-Year plan framework of programmes on
(c) Eighth Five-Year Plan sustainable consumption and production
(d) Tenth Five-Year plan patterns.
Which of the statements given above are
51. According to IPCC's Fifth Assessment correct?
Report (AR5) published in 2014, which of (a) 1 only
the following countries has the highest (b) 2 only
cumulative contribution to global CO2 (c) Both 1 and 2
emissions? (d) Neither 1 nor 2
(a) China
(b) USA 55. Which of the following changes were
(c) India introduced in pursuance of the New
(d) Russia Industrial Policy, 1991 in India?
1. No licences were required for setting up
52. Consider the following initiatives: new industrial units and expansion of the
1. Perform Achieve and Trade Scheme. existing units in many industries.
2. Market Transformation for Energy 2. The Foreign Investment Promotion
Efficiency. Board (FIPB) was set up.
3. Energy Efficiency Financing Platform. 3. The private sector’s participation in the
4. Framework for Energy Efficient economy increased.
Economic Development. How many of the statements given above are
How many of the above are the initiatives correct?
under the National Mission for Enhanced (a) Only one
Energy Efficiency (NMEEE)? (b) Only two
(a) Only one (c) All three
(b) Only two (d) None
(c) Only three
(d) All four

https://upscpdf.com/ https://upscpdf.com/
https://t.me/visioniastestseries2024
https://upscpdf.com/ https://upscpdf.com/

10

56. Consider the following statements with Which of the statements given above are
reference to Leptospirosis: incorrect?
1. It is a viral disease caused by the virus of (a) 1 and 2 only
the genus Leptospira. (b) 2 and 3 only
2. The disease can spread through the urine (c) 1 and 3 only
of infected animals. (d) 1, 2 and 3
Which of the statements given above are
correct? 59. Consider the following statements with
(a) 1 only reference to the Bank for International
(b) 2 only Settlements:
(c) Both 1 and 2 1. It was established post-Second World
(d) Neither 1 nor 2 War as an international financial
institution offering banking services for
57. Consider the following statements with national central banks.
reference to the Genetic Engineering 2. India was the founding member of the
Appraisal Committee: Bank.
1. It functions under the administrative Which of the statements given above are
Control of the Ministry of Science and correct?
Technology. (a) 1 only
2. GEAC is chaired by the Minister of (b) 2 only
Science and Technology. (c) Both 1 and 2
3. The committee can take punitive actions (d) Neither 1 nor 2
under the Environment Protection Act.
How many of the statements given above are 60. Consider the following statements with
correct? reference to the Extended Credit Facility
(a) Only one (ECF):
(b) Only two 1. The ECF is one of the facilities under the
(c) All three Poverty Reduction and Growth Trust
(d) None (PRGT).
2. There are no conditionalities attached to
58. Consider the following statements regarding availing the Extended Credit Facility.
the socio-economic and ecological impacts of Which of the statements given above are
the Green Revolution in India: correct?
1. The Green Revolution decreased income (a) 1 only
disparities among farmers in India. (b) 2 only
2. The cropping patterns in the Green (c) Both 1 and 2
Revolution substantially increased the (d) Neither 1 nor 2
cultivation of pulses, oilseeds, maize, and
barley. 61. Consider the following:
3. The Green Revolution had an adverse 1. Biodiversity Protection.
ecological impact. 2. Renewable Energies
3. Industrial Pollution Control
4. Water Sanitation

https://upscpdf.com/ https://upscpdf.com/
https://t.me/visioniastestseries2024
https://upscpdf.com/ https://upscpdf.com/

11

Investments in how many of the above areas 64. With reference to Export Credit Guarantee
will be considered as Green Finance? Corporation of India, consider the following
(a) Only one statements:
(b) Only two 1. It provides Credit Risk Insurance and
(c) Only three related services for exports.
(d) All four 2. It assists exporters in recovering bad
debts.
62. Consider the following pairs: 3. It facilitates the availability of adequate
Species IUCN Status bank finance to the Indian exporters.
1. Himalayan Critically How many of the above statements are
Brown Bear Endangered correct?
2. Jerdon’s Least concerned (a) Only one
Courser (b) Only two
3. Mountain Vulnerable (c) All three
Pygmy Possum (d) None
How many of the pairs given above are
correct? 65. Which of the following factors contributed to
(a) Only one India’s Balance of Payment Crisis of 1991?
(b) Only two 1. Higher oil prices due to the Gulf War
(c) All three 2. High Inflation
(d) None 3. Low Forex
4. A sharp decline in the private remittances
63. Consider the following statements regarding from overseas Indian workers
cropping patterns in Indian agriculture: Select the correct answer using the code
1. The multiplicity of cropping systems in given below:
India is attributed to the prevalence of (a) 1 and 2 only
rainfed agriculture and the socio- (b) 2, 3 and 4 only
economic conditions of the farming (c) 1 and 3 only
community. (d) 1, 2, 3 and 4
2. Increased demand for food has led to 66. With reference to second-generation
crop intensification in India. economic reforms in India, consider the
3. The choice of crops and cropping following statements:
systems at the farmer's level is primarily 1. It led to the dismantling of the
influenced by soil and climatic Administered Price Mechanism (APM).
parameters. 2. It leads to a reduction of the state
Which of the statements given above are government's role in the economic
correct? reform process.
(a) 1 and 2 only Which of the statements given above are
(b) 2 and 3 only correct?
(c) 1 and 3 only (a) 1 only
(d) 1, 2 and 3 (b) 2 only
(c) Both 1 and 2
(d) Neither 1 nor 2

https://upscpdf.com/ https://upscpdf.com/
https://t.me/visioniastestseries2024
https://upscpdf.com/ https://upscpdf.com/

12

67. Consider the following pairs: 70. Which one of the following statements
Places Country regarding Pradhan Mantri Kaushal Vikas
1. Arkadag Turkmenistan Yojana is incorrect?
2. Kaas Plateau India (a) It aims to promote skill development in
3. Khat-ar-shnong China the country by providing short-duration
How many of the pairs given above are skill training.
correctly matched? (b) It encourages standardisation of the
(a) Only one Certification process and puts in place
(b) Only two the foundation for creating a registry of
(c) All three skills.
(d) None (c) It is a flagship scheme of the Ministry of
Micro, Small and Medium Enterprises.
68. Which one of the following statements about (d) The scheme imparts training in soft
disinvestment is incorrect? skills, entrepreneurship, financial and
(a) Disinvestment is the process of selling digital literacy.
ownership of a company by the
government. 71. Consider the following statements with
(b) Disinvestment is a tool of resource reference to the Digital Publisher Content
mobilisation for budgetary needs. Grievances Council (DPCGC):
(c) Strategic disinvestment is the sale of a 1. It has been set up under the aegis of
substantial portion of the Government Internet and Mobile Association of India.
shareholding. 2. It is registered by the Ministry of
(d) The Ministry of Finance solely identifies Information & Broadcasting as a Level-
the PSUs to be strategically Disinvested. II Self-Regulatory Body for publishers of
online curated content.
69. Consider the following statements with Which of the statements given above are
reference to the Anusandhan National correct?
Research Foundation Bill, 2023: (a) 1 only
1. It repeals the Science and Engineering (b) 2 only
Research Board Act, of 2008. (c) Both 1 and 2
2. The Foundation will be financed only (d) Neither 1 nor 2
through grants and loans from the central
government. 72. Consider the following statements about the
3. The Governing Board of the foundation National Investment Fund (NIF):
will be headed by the Prime Minister of 1. It was constituted in 2005 into which the
India. proceeds from the disinvestment of
How many of the statements given above are Central Public Sector Enterprises were to
correct? be channelised.
(a) Only one 2. The NIF is maintained as an integral part
(b) Only two of the Consolidated Fund of India.
(c) All three
(d) None

https://upscpdf.com/ https://upscpdf.com/
https://t.me/visioniastestseries2024
https://upscpdf.com/ https://upscpdf.com/

13

Which of the statements given above are 75. Which one of the following statements
correct? regarding MSME (Micro, Small and Medium
(a) 1 only Enterprises) is incorrect?
(b) 2 only (a) The MSMEs are classified into
(c) Both 1 and 2 manufacturing and service enterprises as
(d) Neither 1 nor 2 per the MSME Act, 2006.
(b) The sector has huge potential to address
73. Which of the following were the measures the unemployment problem.
adopted in India for the promotion of (c) Loans up to Rs.1 crores in some cases are
globalisation during the economic reforms? to be approved within 59 minutes to
1. Reduction of import duties. encourage MSMEs.
2. Encouragement of foreign investment. (d) The share of MSME manufacturing
3. Licensing regimes for various industries output in all India Manufacturing output
were introduced. is around 65% during the last five years.
4. Encouragement of foreign technology
agreements. 76. Which of the following statements with
Select the correct answer using the code given regard to the Indian Tourism Industry are
below: correct?
(a) 1 and 2 only 1. The Ministry of Tourism has designated
(b) 2, 3 and 4 only the Year 2023 as the ‘Visit India’ Year.
(c) 1, 2 and 4 only 2. India allows 100% FDI under the
(d) 3 and 4 only automatic route in the Tourism sector.
3. The PRASAD scheme focuses on
74. Consider the following statements with improving the religious tourism
reference to the Central Bureau of experience in India.
Investigation: Select the correct answer using the code
1. It has been set up under the Delhi Special given below:
Police Establishment (DSPE) Act, 1946. (a) 1 and 2 only
2. It must mandatorily obtain the consent of (b) 2 and 3 only
the state government before beginning to (c) 1 and 3 only
investigate a crime in the state. (d) 1, 2 and 3
3. The first state to withdraw consent was
Mizoram in 2015. 77. Which of the following policies were taken
How many of the statements given above are under the External Sector Reforms of 1991-
incorrect? 2000?
(a) Only one 1. Abolishing quantitative restrictions on
(b) Only two import.
(c) All three 2. Announcing full capital account
(d) None convertibility.
3. Promulgation of a liberal Foreign
Exchange Management Act.

https://upscpdf.com/ https://upscpdf.com/
https://t.me/visioniastestseries2024
https://upscpdf.com/ https://upscpdf.com/

14

Select the correct answer using the codes 2. The National Rail plan aims to increase
given below: the modal share of Railways in freight to
(a) 1 and 2 only 45%.
(b) 2 and 3 only 3. 100% FDI under the automatic route is
(c) 1 and 3 only allowed in the construction, operation
(d) 1, 2 and 3 and maintenance of Mass Rapid
Transport Systems.
78. Consider the following statements about Which of the statements given above are
Swiss Challenge model of Investment: correct?
1. It can be used in Public Private (a) 1 and 3 only
Partnerships (PPP) as well as non-PPP (b) 2 and 3 only
Projects. (c) 1 and 2 only
2. The project is always awarded to the (d) 1, 2 and 3
original bidder.
81. Consider the following statements with
3. The Swiss Challenge model for
reference to disinvestments in the economy:
infrastructure development has not yet
1. Disinvestment is a process of selling
been used in India.
government equity in public sector
How many of the statements given above are
enterprises.
correct?
2. Privatisation refers to the economic
(a) Only one
policies that promote the expansion of
(b) Only two
the private sector.
(c) All three
Which of the statements given above are
(d) None
incorrect?
(a) 1 only
79. Which one of the following statements
(b) 2 only
regarding Foreign Direct Investment (FDI) in
(c) Both 1 and 2
India is incorrect?
(d) Neither 1 nor 2
(a) Investment climate in India has improved
considerably since the opening up of the 82. Consider the following statements with
economy in 1991. reference to the Paris agreement:
(b) Singapore emerged as the top source for 1. It reaffirms that developed countries
FDI equity inflows into India in FY should take the lead in providing
2022-23. financial assistance to less developed
(c) Under the Automatic Route, the non- countries.
resident investor does not require any 2. It ignores the climate-related capacity-
approval from the Government of India building aspects for developing
for the investment. countries.
(d) FDI is prohibited in the Lottery Business. Which of the statements given above are
correct?
80. Consider the following statements with (a) 1 only
reference to Indian Railways: (b) 2 only
1. India has the largest railway system in the (c) Both 1 and 2
world. (d) Neither 1 nor 2

https://upscpdf.com/ https://upscpdf.com/
https://t.me/visioniastestseries2024
https://upscpdf.com/ https://upscpdf.com/

15

83. Which of the following are among the four 2. The unorganised workers suffer from
pillars of the Make in India initiative? cycles of excessive seasonality of
1. New processes employment.
2. New infrastructure Which of the statements given above are
3. New sectors correct?
4. New mindset (a) 1 only
Select the correct answer using the code (b) 2 only
given below: (c) Both 1 and 2
(a) 1 and 3 only (d) Neither 1 nor 2
(b) 2, 3 and 4 only
(c) 1, 2 and 4 only 87. Consider the following statements with
(d) 1, 2, 3 and 4 regard to the Stand-Up India scheme:
1. The Stand-Up India Scheme was
84. Which one of the following best describes an launched to promote entrepreneurship
economy's Forex Reserves? among women, Scheduled Castes(SC)
(a) Gold reserves only and Scheduled Tribes(ST) categories.
(b) Gold reserves and foreign currency 2. The loans under the scheme are available
assets only for both greenfield and brownfield
(c) Foreign currency assets, gold reserves, projects.
SDRs and IMF Reserve Tranche 3. In the case of non-individual enterprises,
(d) Gold reserves, foreign currency assets a maximum of 20% of the shareholding
and SDRs only and controlling stake should be held by
either SC / ST or Women Entrepreneurs.
85. Consider the following statements with How many of the statements given above are
reference to the Euclid mission: correct?
1. It is a planned mission to investigate the (a) Only one
mysteries of dark matter and dark energy. (b) Only two
2. This mission was launched by the (c) All three
European Space Agency. (d) None
Which of the statements given above are
correct? 88. U-DISE+ recently seen in the news is related
(a) 1 only to which of the following?
(b) 2 only (a) A system of online data collection from
(c) Both 1 and 2 the schools
(d) Neither 1 nor 2 (b) Culture awareness scheme of the
Ministry of Culture
86. Consider the following statements with (c) Diaspora engagement model of the
reference to the Unorganised workers in Ministry of External Affairs
India: (d) Skill development programme the
1. The term unorganised worker has been Ministry of Labour & Employment
defined under the Unorganized Workers'
Social Security Act, 2008.

https://upscpdf.com/ https://upscpdf.com/
https://t.me/visioniastestseries2024
https://upscpdf.com/ https://upscpdf.com/

16

89. Consider the following statements regarding Which of the statements given above are
India's foreign exchange reserves: correct?
1. The RBI officially targets a specific (a) 1 and 2 only
exchange rate. (b) 2 and 3 only
2. When RBI buys dollars in the spot (c) 1 and 3 only
market, it injects rupees into the (d) 1, 2 and 3
economy.
Which of the statements given above are 92. Consider the following statements with
correct? reference to the Mission on Advanced and
(a) 1 only High-Impact Research (MAHIR):
(b) 2 only 1. It is jointly launched by the Ministry of
(c) Both 1 and 2 Finance and the Ministry of New and
(d) Neither 1 nor 2 Renewable Energy.
2. Its main aim is to identify emerging
90. Consider the following statements with technologies and areas of future
regard to the Revamped Distribution Sector relevance for the Global Power Sector.
Scheme (RDSS): 3. The Technical Scoping Committee in the
1. The Scheme was launched to improve the MAHIR will be chaired by the
financial position of state Agriculture Chairperson of the Central Electricity
Produce Market Committees (APMCs). Authority.
2. The release of funds under the scheme How many of the statements given above are
has been linked to Results and Reforms. correct?
3. There is no sunset clause in the scheme. (a) Only one
Which of the statements given above are (b) Only two
incorrect? (c) All three
(a) 1 and 2 only (d) None
(b) 1 and 3 only
(c) 2 and 3 only 93. Consider the following statements with
(d) 1, 2 and 3 reference to the INDUS-X:
1. It is jointly launched by France and India.
91. Consider the following statements regarding 2. It aims to expand the strategic technology
currency exchange: partnership and defence industrial
1. The purpose of a fixed exchange rate cooperation between the two
system is to keep a currency's value governments, businesses, and academic
within a narrow band. institutions.
2. A Floating Currency Regime allows a Which of the statements given above are
domestic currency to freely fluctuate correct?
based on market forces. (a) 1 only
3. Managed Exchange Rates involve a (b) 2 only
government's direct or indirect influence (c) Both 1 and 2
on currency values. (d) Neither 1 nor 2

https://upscpdf.com/ https://upscpdf.com/
https://t.me/visioniastestseries2024
https://upscpdf.com/ https://upscpdf.com/

17

94. Which of the following shifts occurred in 97. Consider the following sectors:
India's currency system in the financial year 1. Telecom & Networking Products
1992-93? 2. Drones and Drone Components
(a) Transition from the gold standard to the 3. Pharmaceuticals
fixed currency system 4. Food Products
(b) Introduction of the dual exchange rate How many of the above sectors are eligible
mechanism for the benefits of the production Linked
(c) Adoption of an ideal free-floating Incentive (PLI) Scheme?
exchange rate (a) Only one
(d) Tying the rupee's value to the British (b) Only two
Pound Sterling (c) Only three
(d) All four
95. Under the process of economic reforms, an
Insurance Reforms Committee (IRC) was set 98. Consider the following statements regarding
up in 1993 under the chairmanship of who the capital account in a government's
among the following? economic transactions with external
(a) Narsimham committee economies:
(b) C. Rangrajan committee 1. The capital account tracks a country's
(c) Manmohan Singh committee trade and services balance with other
(d) R. N. Malhotra committee nations.
2. Foreign currency deposits of banks form
96. Consider the following statements regarding part of the capital account.
the current account in the context of the Which of the statements given above are
external sector: correct?
1. Current account includes the transactions (a) 1 only
such as export, import, interest payments, (b) 2 only
private remittances, and transfers. (c) Both 1 and 2
2. India experienced a deficit current (d) Neither 1 nor 2
account for three consecutive years
(2000-03). 99. Consider the following statements with
3. The sustainable level of the current reference to the Commission of Railway
account deficit for India is at 12.5 percent safety:
of the GDP. 1. It works Under the Ministry of Civil
Which of the statements given above are Aviation of the Government of India.
incorrect? 2. The duties of the commissioner of
(a) 1 and 2 only railway safety are mentioned in the
(b) 2 and 3 only Railway Act of 1989.
(c) 1 and 3 only Which of the statements given above are
(d) 1, 2 and 3 correct?
(a) 1 only
(b) 2 only
(c) Both 1 and 2
(d) Neither 1 nor 2

https://upscpdf.com/ https://upscpdf.com/
https://t.me/visioniastestseries2024
https://upscpdf.com/ https://upscpdf.com/

18

100. Consider the following statements regarding


the Balance of Payment (BoP):
1. BoP is a measure of an economy's total
transactions with the outside world in a
given year.
2. Negative BoP necessarily and always
indicates a dangerous situation for the
economy.
3. The BoP is calculated based on the
principles of double-entry bookkeeping.
4. If the forex reserves are insufficient to
cover a negative BoP, it leads to a BoP
crisis.
How many of the statements given above are
correct?
(a) Only one
(b) Only two
(c) Only three
(d) All four

https://upscpdf.com/ https://upscpdf.com/
https://t.me/visioniastestseries2024
https://upscpdf.com/ https://upscpdf.com/

Prelims Wallah Test 2024


Test 14: Economy Part II Synopsis
1. D 26. A 51. B 76. D

2. B 27. C 52. D 77. C

3. C 28. D 53. C 78. A

4. C 29. D 54. C 79. B

5. D 30. B 55. C 80. B

6. D 31. C 56. B 81. D

7. A 32. D 57. A 82. A

8. B 33. C 58. A 83. D

9. C 34. D 59. D 84. C

10. D 35. B 60. A 85. C

11. C 36. D 61. D 86. C

12. D 37. D 62. A 87. A

13. A 38. D 63. D 88. A

14. C 39. A 64. C 89. B

15. A 40. D 65. D 90. B

16. A 41. B 66. A 91. D

17. C 42. B 67. B 92. B

18. D 43. B 68. D 93. B

19. A 44. B 69. B 94. B

20. D 45. C 70. C 95. D

21. D 46. C 71. C 96. B

22. A 47. B 72. A 97. D

23. B 48. B 73. C 98. B

24. A 49. B 74. A 99. C

25. A 50. C 75. D 100. C

https://upscpdf.com/ https://upscpdf.com/
https://t.me/visioniastestseries2024
https://upscpdf.com/ https://upscpdf.com/

1. Ans: D time and three as part-time members)


Exp: appointed and nominated by the
The National Planning Committee was government. The authority is responsible
established in October 1938 by Subash for the regulation, development and
Chandra Bose (the then-president of the supervision of the Indian insurance
Indian National Congress). Its main industry.
objective was to focus on the development
of industries in India. Jawaharlal Nehru The powers and functions of the
chaired the Committee. Authority are:
The formation of the committee was based A. Certification of insurance companies
on the resolutions passed during the B. Protecting the interest of the
Conference of Ministers of Industries. The policyholders
conference recognised that industrialisation C. Adjudication of disputes
was the key to India's economic • Issue to the applicant a certificate
regeneration, poverty reduction, and of registration, renew, modify,
unemployment. Therefore, the Planning withdraw, suspend or cancel such
Committee was appointed to address these registration;
issues. • Protection of the interests of the
policyholders in matters
2. Ans: B concerning assigning of policy,
Exp: nomination by policyholders,
Statement 1 is correct: India achieved full insurable interest, settlement of
convertibility in its current account on insurance claim, surrender value of
August 19, 1994. It means that the full the policy and other terms and
amount of the foreign exchange required by conditions of contracts of
someone for current purposes will be made insurance;
available to him at the official exchange • specifying requisite qualifications,
rate, and there could be an unprohibited code of conduct and practical
outflow of foreign exchange. training for intermediary or
Statement 2 is incorrect: Individuals in insurance intermediaries and
India are allowed to invest in foreign agents.
assets, shares, etc., but there is a limit of • specifying the code of conduct for
USD 2,50,000 per annum. It is not an surveyors and loss assessors;
unlimited amount. Indian corporations are
allowed full convertibility in the automatic 4. Ans: C
route of up to USD 500 million in overseas Exp:
ventures. The International Monetary Fund (IMF)
Statement 3 is incorrect: After the came up in 1944, and its articles came into
recommendations of the S.S. Tarapore force on December 27, 1945, with the main
Committee (1997) on Capital Account functions as:
Convertibility, India has been moving in • The exchange rate regulation.
the direction of allowing full convertibility • Purchasing short-term foreign
in this account, but with the required currency liabilities of the
precautions. India is still a country of member nations from around
partial convertibility (40:60) in the the world,
capital account. • Allotting special drawing rights
(SDRs) to the member nations
3. Ans: C • The bailout for the member
Exp: economies in the event of any BoP
The Insurance Regulatory and crisis.
Development Authority (IRDA) was set up • To facilitate international
in 2000 (the Act was passed in 1999) with monetary cooperation,
one chairman and five members (two as full

https://upscpdf.com/ https://upscpdf.com/
https://t.me/visioniastestseries2024
https://upscpdf.com/ https://upscpdf.com/

• To promote exchange rate stability The conventions, for which the GEF serves
and orderly exchange as the financial mechanism, provide broad
arrangements, strategic guidance to the two governing
• To assist in the establishment of a bodies of the GEF: the GEF Council and
multilateral system of payments the GEF Assembly. The GEF Council
and the elimination of foreign converts this broad guidance into
exchange restrictions; operational criteria (guidelines) for GEF
• To assist member countries by projects.
temporarily providing financial
resources to correct mal- 6. Ans: D
adjustment in their balance of Exp:
payments (BoPs). The growth in the insurance sector is
Reference: Ramesh Singh, Chapter 16, internationally measured based on the
standard of insurance penetration.
5. Ans: D Statement 1 is incorrect: Insurance
Exp: penetration is defined as the ratio of
Context: At a recent Global Environment premiums underwritten in a given year
Facility (GEF) council meeting in Brazil, to the gross domestic product (GDP). It
the governing body approved the is one of the parameters used to assess the
disbursement of $1.4 billion to accelerate level of development of the insurance
efforts to tackle the climate, biodiversity, sector in a country. In 2021-22, insurance
and pollution crises. penetration in India stood
The Global Environment Facility (GEF), at 4.2 per cent (life insurance penetration at
established on the eve of the 1992 Rio Earth 3.2 percent and general insurance,
Summit, is a catalyst for action on the including health, at 1 percent). This is quite
environment — and much more. Through low when compared to other developed
its strategic investments, the GEF works economies such as the US and Canada
with partners to tackle the planet’s biggest (11.4 per cent) and advanced Europe,
environmental issues. It has a unique Middle-east and African regions.
governing structure organised around an Statement 2 is incorrect: Insurance
Assembly, the Council, the Secretariat, 18 density is another well-recognised
agencies, a Scientific and Technical benchmark and is defined as the ratio of
Advisory Panel, and the Evaluation Office. premiums underwritten in a given year
The GEF serves as a financial mechanism to the total population (measured in US
for several environmental conventions. dollars for convenience of comparison).
The GEF provides funding to assist In India, insurance penetration has grown
developing countries in meeting the from 2.3 per cent (life 1.8 per cent and non-
objectives of international environmental life 0.7 per cent) in 2000 to 3.9 per cent (life
conventions. The GEF serves as a 3.1 per cent and non-life 0.8 per cent) in
"financial mechanism" for five 2013. In the comparable period, insurance
conventions: density has improved from US$ 11 to US$
• United Nations Convention on 52.
Biological Diversity (CBD)
• United Nations Framework 7. Ans: A
Convention on Climate Change Exp:
(UNFCCC) The Global Financial Stability Report
• Stockholm Convention on (GFSR) is a publication by the
Persistent Organic Pollutants International Monetary Fund (IMF) that
(POPs) provides an assessment of the global
• UN Convention to Combat financial system and potential risks to
Desertification (UNCCD) financial stability. The report is typically
• Minamata Convention on released twice a year, and it analyses key
Mercury.

https://upscpdf.com/ https://upscpdf.com/
https://t.me/visioniastestseries2024
https://upscpdf.com/ https://upscpdf.com/

developments in the global financial the Post-War Reconstruction Committee


markets. of the Indian Trade Union. The plan was
Other IMF Reports: based on Marxist socialism and advocated
• World Economic Outlook the need to provide the people with the
• Global financial stability report ‘basic necessities of life’.
• Fiscal Monitor Pair 4 is correct: Bombay Plan was the
popular title of ‘A Plan of Economic
World Bank Reports Development for India’, which was
• World Development Report prepared by a cross-section of India’s
• International Debt Statistics leading capitalists. The eight capitalists
• Ease of doing Business Report involved in this plan were Purshottamdas
• Global Economic Prospects Thakurdas, J.R.D. Tata, G.D. Birla, Lala
Sri Ram, Kasturbhai Lalbhai, A.D. Shroff,
8. Ans: B Ardeshir Dalal and John Mathai. The Plan
Exp: was published in 1944— 45. Out of these
Though the planned economic eight industrialists, Purshottamdas
development in India began in 1951 with Thakurdas was one among the 15 members
the inception of the First Five-Year Plan, of the National Planning Committee (1938)
theoretical efforts had begun much earlier, The remaining three J.R.D. Tata, G.D. Birla
even prior to independence. and Lala Sri Ram were the members of the
Pair 1 is correct: After the reports of the sub-committees (29 in total) of the National
National Planning Committee (NPC) were Planning Committee.
published and the government was set to go
for the Five-Year plans, a lone blueprint for 9. Ans: C
the planned development of India was Exp:
formulated by the famous socialist The Economic Intelligence Unit’s (EIU)
leader Jaiprakash Narayan— the Global Liveability Index quantifies the
Sarvodaya Plan published in January challenges presented to an individual’s
1950. Central ideas of the plan were highly lifestyle and standard of living in 173
similar to the Gandhian Plan, like emphasis cities worldwide. This year’s average
on agriculture, agro-based small and liveability score has risen to a 15-year high,
cottage industries, self-reliance and almost with a return to order after the COVID-19
no dependence on foreign capital and pandemic and better healthcare and
technology, land reforms, self-dependent education in developing countries.
villages and decentralised participatory The themes included in this year's report
form of planning and economic progress, to are:
name the major ones. • Which cities have the best and
Pair 2 is incorrect: Espousing the spirit of worst living conditions and quality
Gandhian economic thinking, Sriman of life around the world?
Narayan Agarwal formulated this plan • A return to order after the COVID-
in 1944. This plan laid more emphasis on 19 pandemic.
agriculture. Even if he referred to • Developing countries make
industrialisation, it was to the level of improvements to liveability
promoting cottage and village-level • How the war in Ukraine and the
industries, unlike the NPC and the Bombay resulting economic and political
Plan, which supported a leading role for the disruptions continue to affect
heavy and large industries. The plan livability across major cities
articulated a ‘decentralised economic
structure’ for India with ‘self-contained 10. Ans: D
villages’. Exp:
Pair 3 is incorrect: In 1945, the Peoples The terms Extended Fund Facility (EFF)
Plan was formulated by the radical and Financial Transactions Plan (FTP)
humanist leader M.N. Roy, chairman of

https://upscpdf.com/ https://upscpdf.com/
https://t.me/visioniastestseries2024
https://upscpdf.com/ https://upscpdf.com/

are associated with the International


Monetary Fund (IMF).
• Extended Fund Facility (EFF):
The Extended Fund Facility (EFF)
is a service provided by the IMF to
its member countries which
authorizes them to raise any
amount of foreign exchange from it
to fulfil their BoP crisis but on the
conditions of structural reforms in Map: Israel and Bordering Countries.
the economy put by the body. It is
the first agreement of its kind. 12. Ans: D
India signed this agreement with Exp:
the IMF in the financial year 1981– Statement 1 is incorrect: The insurance
82. sector was opened up for private sector
• Financial Transactions Plan in 2000 after the enactment of the
(FTP): It is the mechanism of the Insurance Regulatory and Development
IMF through which it Authority Act, 1999 (IRDA Act, 1999),
finances/repays its operations. which had increased the Foreign Direct
Member nations contribute Investment (FDI) limit to 26% as the sector
money to it from their 'quota was facing a severe shortage of funds. The
resources’ on which they get IRDAI was incorporated as a statutory
'interest'.. body in April 2000. The key objectives of
11. Ans: C the IRDAI include the promotion of
Exp: competition so as to enhance customer
Israel is a country in the Middle East, satisfaction through increased consumer
located at the eastern end of the choice and lower premiums while ensuring
Mediterranean Sea. It is bounded by: the financial security of the insurance
• In the North by Lebanon market.
• To the northeast by Syria Statement 2 is incorrect: Govt., in
• To the east and southeast by February 2015, raised the foreign
Jordan investment ceiling in the insurance sector
• To the southwest by Egypt from 26% to 49% under the automatic
• To the west by the route, and then in 2021, it has again been
Mediterranean Sea. increased to 74%.
Jerusalem is the seat of government and the
proclaimed capital, although the latter 13. Ans: A
status has not received wide international Exp:
recognition. The Reserve Bank of India (RBI) has
Israel is a small country with a relatively come up with a Financial Inclusion
diverse topography, consisting of a lengthy Dashboard named Antardrishti, a
coastal plain, highlands in the north and platform that will provide insights to
central regions, and the Negev desert in the assess and monitor the progress of
south. Running the length of the country financial Inclusion. This facility will also
from north to south along its eastern border enable us to gauge the extent of financial
is the northern terminus of the Great Rift exclusion at granular levels across the
Valley. The State of Israel is the only country so that such areas can be addressed.
Jewish nation in the modern period, and the The Dashboard is presently intended for
region that now falls within its borders has Internal use in the RBI and will further
a lengthy and rich history that dates from facilitate greater financial inclusion
pre biblical times. through a multi-stakeholder approach.
Saudi Arabia does not border Israel.

https://upscpdf.com/ https://upscpdf.com/
https://t.me/visioniastestseries2024
https://upscpdf.com/ https://upscpdf.com/

14. Ans: C an economy but at risks independent


Exp: sovereign decisions, too.
The Multilateral Investment Guarantee
Agency (MIGA), set up in 1988, 15. Ans: A
encourages foreign investment in Exp:
developing economies by offering Statement 1 is correct: The disease mpox
insurance (guarantees) to foreign private (formerly monkeypox) is caused by the
investors against losses caused by non- monkeypox virus (commonly
commercial (i.e., political) risks such as abbreviated as MPXV), an enveloped
currency transfer, expropriation, war and double-stranded DNA virus of the
civil disturbance. India became a member Orthopoxvirus genus in the Poxviridae
of MIGA in January 1994. family, which includes variola, cowpox,
vaccinia and other viruses. The two genetic
International Finance Corporation clades of the virus are clades I and II.
(IFC) was set up in 1956 and is also known Statement 2 is correct: Person-to-person
as the private arm of the World Bank. It transmission of mpox can occur through
lends money to private sector companies in direct contact with infectious skin or
its member nations. The interest rate other lesions, such as in the mouth or on
charged is commercial but comparatively the genitals. This includes contact which
low. India is one of the founding is:
members of the IFC. • face-to-face (talking or breathing)
• skin-to-skin (touching or
The International Development Agency vaginal/anal sex)
(IDA), also known as the soft window of • mouth-to-mouth (kissing)
the World Bank, was set up in 1960 with • mouth-to-skin contact (oral sex or
the basic aim of developing infrastructural kissing the skin)
support among the member nations and • respiratory droplets or short-range
long-term lending for the development of aerosols from prolonged close
economic services. India is one of the contact
founding members of IDA. The virus then enters the body through
broken skin, mucosal surfaces (e g oral,
The International Bank for pharyngeal, ocular, genital, anorectal), or
Reconstruction and Development via the respiratory tract. Mpox can spread
(IBRD) is the oldest of the WB institutions to other members of the household and to
that started functioning in 1945 in the area sex partners.
of reconstruction of the war-ravaged Statement 3 is incorrect: The goal of
regions (World War II) and later for the treating mpox is to take care of the rash,
development of the middle-income and manage pain and prevent complications.
credit-worthy poorer economies of the Early and supportive care is important to
world. India is one of the founding help manage symptoms and avoid further
members of IBRD. problems. Getting a mpox vaccine can
The International Centre for Settlement help prevent infection. The vaccine
of Investment Disputes (ICSID), set up in should be given within 4 days of contact
1966, is an investment dispute settlement with someone who has mpox (or within up
body whose decisions are binding on the to 14 days if there are no symptoms). It is
parties. It was established under the 1966 recommended for people at high risk to get
Convention on the Settlement of Investment vaccinated to prevent infection with mpox,
Disputes between States and Nationals of especially during an outbreak.
Other States.
India is not a member of ICSID. It is 16. Ans: A
believed that being a signatory to it Exp:
encourages foreign investment flows into Statement 1 is correct: The Five-Year
Plans were centralised and integrated

https://upscpdf.com/ https://upscpdf.com/
https://t.me/visioniastestseries2024
https://upscpdf.com/ https://upscpdf.com/

national economic programs. The first Statement 1 is incorrect: AIIB offers


such plan was implemented in the Soviet sovereign (government) and non-sovereign
Union in 1928 by Joseph Stalin. Since (private) financing for sound and
then, countries such as China, Bhutan, sustainable projects in energy and power,
Vietnam, South Korea, Argentina, transportation and telecommunications,
Romania and Ethiopia have also rural infrastructure and agriculture
implemented Five-Year Plans. development, water supply and sanitation,
Statement 2 is incorrect: Economic and environmental protection, urban
social planning is a concurrent subject. development and logistics.
Also, while framing the ‘Union’, ‘State’ Statement 2 is incorrect: Membership in
and ‘Concurrent’ lists, allocating subjects AIIB shall be open to members of the
and other provisions, the Constitution vests International Bank for Reconstruction and
power in the Union to ensure coordinated Development (IBRD) or the Asian
development in essential fields of activity Development Bank (ADB). China has the
while preserving the initiative and authority highest voting power of 27.5% in the Bank,
of the states in the spheres allotted to them. while India has 7.9% voting power.

17. Ans: C 19. Ans: A


Exp: Exp:
The life insurance business/industry in Statement 1 is correct: The National
the country was nationalised by the Consumer Disputes Redressal
Government of India in 1956 and a fully Commission (NCDRC), is a quasi-
government-owned company, the Life judicial commission in India which was
Insurance Corporation of India (LIC) was set up in 1988 under the Consumer
set up (at that time 245 Indian and foreign Protection Act of 1986. Its head office is
companies were playing in the life segment in New Delhi. The Commission is headed
of insurance). by a sitting or a retired Judge of the Hon'ble
In 1971, the government nationalised the Supreme Court of India or a sitting or a
private sector companies (107 Indian and retired Chief Justice of an Hon'ble High
foreign companies) playing in the general Court, in terms of Rule 3(12)(a) of the
insurance segment, and a government Tribunal(Conditions of Service) Rules,
company, the General Insurance 2021.
Corporation of India (GIC), was formed Statement 2 is incorrect: Section 23 of the
in 1972. Consumer Protection Act, 1986, provides
The Insurance Regulatory and that any person aggrieved by an order of
Development Authority (IRDA) was set NCDRC may prefer an Appeal against
up in 2000 (the Act was passed in 1999) such order to the Supreme Court of
with one chairman and five members (two India within a period of 30 days.
full-time and three part-time members) Section 21 of the Consumer Protection Act,
appointed and nominated by the 1986 posits that the National Commission
government. The authority is responsible shall have jurisdiction to entertain a
for the regulation, development and complaint valued more than two crores and
supervision of the Indian insurance also have Appellate and Revisional
industry. jurisdiction from the orders of State
Commissions or the District fora, as the
18. Ans: D case may be.
Exp:
The Asian Infrastructure Investment 20. Ans: D
Bank (AIIB), established in 2016 and Exp:
headquartered in Beijing, is a multilateral Statement 1 is incorrect: The Asian
financial institution founded to bring Development Bank (ADB), with an
countries together to address the daunting international partnership of its member
infrastructure needs across Asia. countries, was established in 1966 and

https://upscpdf.com/ https://upscpdf.com/
https://t.me/visioniastestseries2024
https://upscpdf.com/ https://upscpdf.com/

has headquarters at Manila, the


Philippines. The Bank is engaged in
promoting the economic and social
progress of its developing member
countries in the Asia-Pacific region.
Its principal functions are as follows
• To make loans and equity
investments for the economic and
social advancement of its
developing member countries;
• To provide technical assistance for 22. Ans: A
the preparation and execution of Exp:
development projects and Statement 1 is correct: With the Partition
programmes and advisory services; as a backdrop, the country reeling with the
• To respond to the requests for influx of refugees, severe food shortage
assistance in coordinating and mounting inflation, the First Five-
development policies and plans in Year Plan was introduced in 1951. The
developing member countries, and plan was based on the Harrod–Domar
• To respond to the requests for model, which suggested that growth was
assistance and coordinate the dependent on two things. First, a high level
development policies and plans of of savings since higher savings enabled
developing member countries. greater investment and second, a low
Statement 2 is incorrect: India is a capital-output ratio that ensured efficient
founding member of ADB. However, India investment and a higher growth rate.
is not the largest shareholder of Statement 2 is incorrect: It focused
ADB. As of 31 December 2022, ADB’s primarily on the development of the
five largest shareholders are Japan and the primary sector, specifically agriculture
United States (each with 15.6% of total and irrigation. The plan aimed at
shares), the People’s Republic of China improving the country’s food production
(6.4%), India (6.3%), and Australia (5.8%). levels to reduce dependence on imports and
Statement 3 is incorrect: Membership in achieve self-sufficiency. The secondary
AIIB shall be open to members of the sector, which includes industries, was not
International Bank for Reconstruction and the primary focus during this period.
Development (IBRD) or the Asian If the First Five-Year Plan focused on
Development Bank (ADB). agriculture and energy, the Second Five-
Year Plan focused on the development of
the public sector and rapid industrialisation.
21. Ans: D Drafted by statistician P.C. Mahalanobis,
Exp: the Second Plan was also called the
Brumbrella takes its name from a large Mahalanobis Plan.
pitch covering that was used at Edgbaston
from 1981 to 2001. 23. Ans: B
It’s the latest in the series of innovations Exp:
implemented by England in the recent Statement 1 is correct: Sovereign Gold
past as they attempt to spice up Test Bonds are government securities
cricket. In this, all fielders were placed in denominated in grams of gold. They are
close-catching positions, three on the substitutes for holding physical gold.
offside and three on the leg, as the field Investors have to pay the issue price in cash
setup looked like an umbrella. and the bonds will be redeemed in cash on
maturity. The Bond is issued by the
Reserve Bank on behalf of the Government
of India.

https://upscpdf.com/ https://upscpdf.com/
https://t.me/visioniastestseries2024
https://upscpdf.com/ https://upscpdf.com/

The quantity of gold for which the investor its reserve tranche position (RTP), then it
pays is protected since he receives the becomes a credit tranche that must be
ongoing market price at the time of repaid in three years with interest.
redemption/ premature redemption. The Initially, member nations’ reserve tranches
SGB offers a superior alternative to holding are normally 25% of their quota. However,
gold in physical form. The risks and costs their RTP can change according to any
of storage are eliminated. lending that the IMF does with its holdings
Statement 2 is incorrect: A minor can of the member’s currency.
invest in these bonds. The application on
behalf of the minor has to be made by 25. Ans: A
his/her guardian. Exp:
Persons resident in India as defined under Statement 1 is correct: India is the
the Foreign Exchange Management Act, world’s second-largest producer of
1999 are eligible to invest in SGB. Eligible crude steel, with an output of 125.32 MT
investors include individuals, HUFs, trusts, of crude steel and finished steel
universities, and charitable institutions. production of 121.29 MT in FY23. India’s
Individual investors with subsequent steel production is estimated to grow 4-7%
change in residential status from resident to to 123-127 MT in FY24. The growth in the
non-resident may continue to hold SGB till Indian steel sector has been driven by the
early redemption/maturity. domestic availability of raw materials such
Statement 3 is correct: The Bonds are as iron ore and cost-effective labour.
issued in denominations of one gram of Consequently, the steel sector has been a
gold and in multiples thereof. The major contributor to India's manufacturing
minimum investment in the Bond shall output.
be one gram with a maximum limit of Statement 2 is correct: The easy
subscription of 4 kg for individuals, 4 kg availability of low-cost manpower and the
for Hindu Undivided Family (HUF), and presence of abundant iron ore reserves
20 kg for trusts and similar entities make India competitive in the global setup.
notified by the government from time to India is home to the seventh-largest
time per fiscal year (April – March). In reserves of iron ore in the world.
the case of joint holding, the limit applies In October 2021, the government
to the first applicant. announced guidelines for the approved
speciality steel production-linked incentive
24. Ans: A (PLI) scheme. Under the Union Budget
Exp: 2023-24, the government allocated Rs.
Statement 1 is correct: The IMF is funded 70.15 crore to the Ministry of Steel.
through its members and their quota Statement 3 is incorrect:
contributions. The reserve tranche is Rajasthan is not the largest producer of
basically an emergency account that steel in the country. States with the Highest
IMF members can access at any time Steel-Producing Capacity in India include:
without agreeing to conditions or paying • Odisha
a service fee. In other words, a portion of a • Chhattisgarh
member country’s quota can be withdrawn • Jharkhand
free of charge at its own discretion. • Karnataka
Statement 2 is incorrect: The reserve
tranches that countries hold with the
IMF are considered their facilities of
first resort, meaning they will tap into the
reserve tranche before seeking a formal
credit tranche.
In theory, members can borrow over 100%
of their quota. However, if the amount
being sought by the member nation exceeds

https://upscpdf.com/ https://upscpdf.com/
https://t.me/visioniastestseries2024
https://upscpdf.com/ https://upscpdf.com/

10

The CRA is one of the mechanisms created


by the NDB to provide financial support to
its member countries in case of balance of
payments problems or other financial
crises. The CRA is being funded 41 per
cent by China, 18 per cent each from Brazil,
India, and Russia, and 5 per cent from
South Africa.
CRA is ‘a framework for the provision of
currency swaps in response to actual or
potential short-term balance of payments
pressures.’

28. Ans: D
Exp:
Statement 1 is incorrect: The National
Development Council (NDC) was set up on
August 6, 1952, by a Resolution issued by
Map: Major Steel Producing States of the cabinet secretariat. Hence, It is not a
India constitutional body.
Statement 2 is incorrect: Considering the
26. Ans: A recommendations of the ‘Administrative
Exp: Reforms Commission’, the NDC was
Statement 1 is correct: Stockholm reconstituted and its functions redefined by
International Peace Research Institute a Cabinet Resolution on October 7, 1967.
(SIPRI) is an independent international The reconstituted NDC comprises the
institute dedicated to research into Prime Minister, all Union Cabinet
conflict, armaments, arms control, and Ministers, Chief Ministers of all States and
disarmament. Established in 1966, SIPRI Union Territories and the Members of the
provides data, analysis, and Planning Commission. It is presided over
recommendations, based on open sources, by the Prime Minister of India.
to policymakers, researchers, media, and Statement 3 is incorrect: The National
the interested public. Development Council (NDC) does not
Statement 2 is incorrect: SIPRI was have the power to directly allocate funds to
established on the basis of a decision by states for the execution of centrally-
the Swedish Parliament and receives a sponsored schemes. The allocation of funds
substantial part of its funding in the form of is typically done through the Union Budget,
an annual grant from the Swedish and the NDC serves as a platform for
Government. The Institute also seeks discussion and coordination among the
financial support from other organisations central and state governments regarding
in order to carry out its research. national development policies.
27. Ans: C 29. Ans: D
Exp: Exp:
The Contingent Reserve Arrangement The Government has launched the
(CRA) is related to additional liquidity ‘Inclusiveness and Accessibility Index’ as
protection for member nations of the part of the ‘Sugamya Bharat Abhiyan’ for
New Development Bank (NDB). Persons with disabilities.
The New Development Bank is also known The government launched
as the BRICS Bank, was established by the the ‘Inclusiveness and Accessibility Index’
BRICS (Brazil, Russia, India, China, and to mark the next chapter of its flagship
South Africa) countries to finance Campaign, the ‘Sugamya Bharat
infrastructure and sustainable development Abhiyan’.
projects in emerging economies.

https://upscpdf.com/ https://upscpdf.com/
https://t.me/visioniastestseries2024
https://upscpdf.com/ https://upscpdf.com/

11

The Index was prepared in collaboration • Prime Minister


with The Federation of Indian Chambers of • Chief Justice of India
Commerce and Industry (FICCI). The • Leader of the Opposition
‘Inclusiveness and Accessibility Index’ • Two eminent personalities
helps industries and corporations to The Union Minister of Culture is not a
participate in the Accessible India member of the committee.
Campaign (AIC) by voluntarily
evaluating their readiness for making 31. Ans: C
the workplace accessible for Persons Exp:
with Disabilities. Statement 1 is correct: Central
The Index has been prepared with a holistic Electricity Authority (CEA) is a
approach and may be applied and used by Statutory Body under the Ministry of
organisations irrespective of their size and Power constituted under the erstwhile
scale. Electricity (Supply) Act, 1948 which was
The absorption and recognition of the Index replaced by the Electricity Act, 2003,
by the Corporate Sector and public sector where similar provisions exist. The CEA
organisations will benefit them immensely is responsible for the technical coordination
by: and supervision of programmes and is also
• Being the benchmark. entrusted with a number of statutory
• Take Progressive steps to increase functions. It is headed by a Chairman, who
support. is also ex-officio Secretary to the
• Fully utilise a diverse talent pool. Government of India and comprises six
• Reduce employee turnover, full-time Members of the CEA.
increase employee loyalty and Statement 2 is correct: Section 73 of the
increase the morale and Electricity Act, 2003 empowers the
productivity of other employees in Authority to perform such functions and
the organisation. duties as the Central Government may
• Create a positive brand image. prescribe or direct, and in particular to
• Expand the customer base through advise the Central Government on
new products and services. matters relating to the national
• Enhance shareholder value. electricity policy, formulates short-term
• Above all, the Inclusiveness and and perspective plans for the
Accessibility Index shall promote development of the electricity system,
human dignity and social cohesion and coordinates the activities of the
where all citizens have equal planning agencies for the optimal
access to opportunities to fully utilisation of resources to sub-serve the
realise their potential. interests of the national economy and to
provide reliable and affordable electricity
30. Ans: B for all consumers.
Exp:
Instituted in 1995, the Gandhi Peace Prize 32. Ans: D
is awarded for social, economic, and Exp:
political transformation through non- Insurance in India is a dynamic sector that
violence, to any deserving person/s or encompasses a wide range of products and
institution/s. The award comprises a services to protect individuals, businesses,
citation and an amount of Rs 1 crore and is and assets against various risks. The
open to all persons regardless of insurance industry in India is regulated by
nationality, race, language, caste, creed or the Insurance Regulatory and Development
gender and any association, institution, or Authority of India (IRDAI). The
organisation. government also has implemented the
Insurance Laws (Amendment) Act, 2015.
The awardee is selected by a five-member Statement 1 is incorrect: Properties in
jury comprising the: India can now be insured with a foreign

https://upscpdf.com/ https://upscpdf.com/
https://t.me/visioniastestseries2024
https://upscpdf.com/ https://upscpdf.com/

12

insurer with prior permission of IRDAI, 34. Ans: D


which was earlier to be done with the Exp:
approval of the Central Government. The arguments advanced in support of the
Statement 2 is incorrect: Appeals against declining female LFPR are from both the
the orders of the Insurance Regulatory and supply and demand side are:
Development Authority (IRDAI) are to be Supply side factors:
referred to SAT as the amended law • More women in rural areas are now
provides for any insurer or insurance pursuing higher education has
intermediary aggrieved by any order made delayed their entry into the labour
by IRDAI to prefer an appeal to the market.
Securities Appellate Tribunal (SAT). • The household incomes have risen
in rural areas on account of higher
33. Ans: C wage levels which is dragging
Exp: women out of the labour market.
Statement 1 is correct: Together with the • Cultural factors, social constraints
process of globalisation, world regional and patriarchal norms restricting
forces have also been asserting their power the mobility and freedom of
through different short of alignments—the women.
Fortaleza Declaration of Heads of State • The relatively higher
(late July 2014) from Brazil, Russia, India, responsibilities of unpaid work
China, and South Africa (the BRICS and unpaid care work.
countries) is another such attempt—the
creation of a BRICS Bank, i.e., the New Demand side factors:
Development Bank (NDB). • Absence of job opportunities and
Statement 2 is correct: quality jobs and significant gender
To fulfil the purpose of the bank, It wage gap are restraining factors.
supports projects in both public and • Structural shift away from
private sectors through loans, equity agricultural employment, and
investments, and other tailored increased mechanisation of
instruments. agriculture.
Major highlights of the bank are as given • The decline in animal husbandry,
below: and in urban areas a fall in
• The bank will have an initial international demand for products
subscribed capital of $50 billion— of labour-intensive industries.
equally shared by the five nations.
• The capital base is to be used for 35. Ans: B
funding infrastructure and Exp:
‘sustainable development’ projects Statement 1 is correct: The
in the BRICS countries initially. International Finance Corporation
• Other low and middle-income (IFC) was set up in 1956 and is also
countries will be able to get known as the private arm of the World
funding as time progresses. Bank. It lends money to private sector
• A Contingent Reserve companies of its member nations. The
Arrangement (CRA) of $100 interest rate charged is commercial but
billion is to be created to provide comparatively low. There are many
additional liquidity protection to attractive features of IFC’s lending. It
member nations during balance of finances and provides advice for private-
payments problems. public ventures and projects in partnership
with private investors and, through its
advisory work, helps governments of the
member nations to create conditions that
stimulate the flow of both domestic and
foreign private savings and investment.

https://upscpdf.com/ https://upscpdf.com/
https://t.me/visioniastestseries2024
https://upscpdf.com/ https://upscpdf.com/

13

Statement 2 is incorrect: It focuses on 37. Ans: D


promoting economic development by Exp:
encouraging the growth of productive The functions of the DICGC are governed
enterprises and efficient capital markets in by the provisions of 'The Deposit Insurance
its member countries. It participates in an and Credit Guarantee Corporation Act,
investment only when it can make a special 1961' (DICGC Act) and 'The Deposit
contribution that complements the role of Insurance and Credit Guarantee
market investors as foreign financial Corporation General Regulations, 1961'
investors (FFI). framed by the Reserve Bank of India.
It also plays a catalytic role, stimulating Banks covered by Deposit Insurance
and mobilising private investment in the Scheme -All commercial banks including
developing world by demonstrating that the branches of foreign banks functioning
investments there, too, can be profitable. in India, Local Area Banks, Regional Rural
Banks and cooperative Banks.
36. Ans: D DICGC insures all bank deposits, such
Exp: as saving, fixed, current, recurring, etc.,
Statement 1 is correct: The Second Five- except the following types of deposits.
Year plan was built on a strategy of long- • Deposits of foreign
term development of the economy. The Governments;
draft of this Plan was prepared by P. C. • Deposits of Central/State
Mahalanobis and Nehru, the Prime Governments;
Ministers of the country at the time. This • Inter-bank deposits
strategy is often called the Nehru- • Deposits of the State Land
Mahalanobis growth strategy. Development Banks with the State
Statement 2 is correct: The Plan Focused cooperative banks;
on the rapid industrialisation of the • Any amount due on account of any
economy, particularly heavy & basic deposit received outside India
industries. It advocated huge imports • Any amount which has been
through foreign loans. It was argued that specifically exempted by the
the development of the industrial sector is a corporation with the previous
precondition for the development of approval of the RBI.
agricultural and other sectors. Hence,
during this phase of planning, capital goods 38. Ans: D
industries like iron and steel, heavy Exp:
engineering, machine tools and heavy The "Global Economic Prospects"
chemical industries were given high report is released by the World Bank.
priority. This semi-annual report examines global
Statement 3 is correct: This strategy, economic developments and prospects,
however, had its limitations as it put with a special focus on emerging markets
more emphasis on capital and developing countries.
goods, which resulted in a scarcity of Other reports by the World Bank:
essential commodities. The problem • Global Financial Development
became acute in the later years of the Report
Second FYP when there was food scarcity • International Debt Statistics
due to bad harvest. • Policy Research Reports (PRRs)
Consequently, in the subsequent Plans, • Doing Business
greater attention was given to agriculture. • Commodity Markets Outlook etc.
As opposed to the emphasis on the role of
capital goods, the emphasis was on the role 39. Ans: A
of consumer goods. Exp:
Statement 1 is correct: The Indian
cropping season is actually classified into
two main seasons - kharif and rabi - based

https://upscpdf.com/ https://upscpdf.com/
https://t.me/visioniastestseries2024
https://upscpdf.com/ https://upscpdf.com/

14

on the monsoon. The Kharif season is from 41. Ans: B


July to October during the South- Exp:
West/Summer Monsoon, while the rabi Statement 1 is incorrect: The share of the
season is from October to March (North- agriculture sector in India's GDP has seen a
East/Returning/Winter Monsoon). significant decline since independence. In
Statement 2 is incorrect: The term Zaid the fiscal year 1950–51, agriculture
refers to crops grown between March accounted for around 55.4% of the GDP,
and June, not July and October. while currently, it stands at around 15 %.
Statement 3 is incorrect: The Kharif crops This demonstrates a substantial decrease in
include rice, maize, sorghum, pearl its contribution to the overall economy over
millet/bajra, finger millet/ragi (cereals), the years.
arhar (pulses), soybean, groundnut Statement 2 is correct: Despite the
(oilseeds), cotton, etc. decreasing share in the country's gross
The rabi crops include wheat, barley, income, around 55% of India's population
oats (cereals), chickpea/gram (pulses), still depends on the agriculture sector for
linseed, mustard (oilseeds) etc. their livelihood. It means that around 55
per cent of the population lives with only
40. Ans: D around 18 per cent of the total income of
Exp: the Indian economy.
In recent times, the Government has been Statement 3 is correct: Agriculture is not
making efforts to formalize the economy. only the biggest sector of the economy, but
In this direction, several initiatives have also the biggest private sector. It is the only
been taken, namely— the introduction of profession which still carries no burden of
GST, digitisation of payments, direct individual income tax.
benefit transfer of Statement 4 is incorrect: The productivity
subsidies/scholarships/wages & salaries gap between on-the-field and ideal farm
to bank accounts, opening of Jan Dhan practices has decreased in recent years. As
accounts, extending social security per the recent release of the Gol, the
coverage to more workers. average productivity of rice, wheat and
As a result of these initiatives, an increase pulses, which was 2,202 kg, 2,802 kg and
in formal employment is evident as is 625 kg per hectare in 2007— 08, increased
shown through multiple data sources: to 2,346 kg, 3,026 kg and 649 kg per
• As per the Annual Survey of hectare during 2011— 13.
Industries (ASI), there has been
an increase in employment in the 42. Ans: B
‘organised’ manufacturing Exp:
sector. The National Land Records
• To indicate the extent of Modernization Programme (NLRMP),
formalisation of the workforce by started in 2008, aims at updating and
extending coverage of organised digitising land records by the end of the
social security, the Government, 12th Plan. Eventually, the intent is to move
since September 2017, published from the presumptive title (where
the monthly payroll data indicating registration of a title does not imply the
the number of new subscribers who owner’s title is legally valid) to a
have availed benefits under three conclusive title (where it does).
major social security schemes, Important points related to this process may
namely— Employees’ Provident be summarised as follows:
Fund (EPFO); Employees’ State 1. Digitisation will help enormously in
Insurance Scheme (ESIC); and the lowering the costs of land transactions,
National Pension Scheme (NPS). while conclusive title will eliminate
legal uncertainty and the need to use
the government as an intermediary for
acquiring land so as to ‘cleanse’ title.

https://upscpdf.com/ https://upscpdf.com/
https://t.me/visioniastestseries2024
https://upscpdf.com/ https://upscpdf.com/

15

2. Given the importance of this Developing States (SIDS), and African


programme, its rollout in various states States.
needs to be accelerated— easier and Statement 3 is incorrect: The Stockholm
quicker land transactions will Convention on Persistent Organic
especially help small and medium Pollutants is a global treaty to protect
enterprises that do not have legal human health and the environment from
support or management. chemicals that remain intact in the
environment for long periods, become
43. Ans: B widely distributed geographically,
Exp: accumulate in the fatty tissue of humans
The Fourth Five-Year plan period was and wildlife, and have harmful impacts on
1969— 74. The Plan was based on the human health or on the environment. The
Gadgil strategy with a special focus on the Global Environment Facility finances
ideas of growth with stability and progress the Stockholm Convention on Persistent
towards self-reliance. Droughts and the Organic Pollutants (POPs). The GEF is
Indo-Pak War of 1971— 72 led the also a Financial Mechanism for four other
economy to capital diversions, creating a major international environmental
financial crunch for the Plan. conventions:
The main emphasis was on the growth • Minamata Convention on Mercury
rate of agriculture to enable other • United Nations Convention on
sectors to move forward. The first two Biological Diversity (UNCBD)
years of the plan saw record production. • United Nations Convention to
The last three years did not measure up due Combat Desertification (UNCCD)
to poor monsoon. Implementation of • United Nations Framework
Family Planning Programmes was Convention on Climate Change
among the major targets of the Plan. (UNFCCC)

44. Ans: B 45. Ans: C


Exp: Exp:
Statement 1 is correct: The Green Climate Countries across the globe committed to
Fund was established by 194 countries create a new international climate
party to the UN Framework Convention on agreement by the conclusion of the U.N.
Climate Change in 2010. It is designed as Framework Convention on Climate
an operating entity of the Convention’s Change (UNFCCC) Conference of the
financial mechanism and is headquartered Parties (COP21) in Paris in December
in the Republic of Korea. It is governed by 2015. In preparation, countries have agreed
a 24 Board member Board, representing to publicly outline what post-2020 climate
countries, and receives guidance from the actions they intend to take under a new
Conference of the Parties to the Convention international agreement, known as their
(COP). Created by the United Nations Intended Nationally Determined
Framework Convention on Climate Contributions (INDCs).
Change (UNFCCC), the Fund aims to India had submitted its Intended Nationally
support a paradigm shift in the global Determined Contribution (INDC) to the
response to climate change. United Nations Framework Convention on
Statement 2 is correct: The fund allocates Climate Change during October 2015. It
its resources to low-emission and climate- has been revised and approved by the
resilient projects and programmes in Cabinet during August 2022.
developing countries. The Fund pays Salient features of India's INDC:
particular attention to the needs of societies • To adopt a climate-friendly and a
that are highly vulnerable to the effects of cleaner path than the one followed
climate change, in particular Least hitherto by others at corresponding
Developed Countries (LDCs), Small Island level of economic development.

https://upscpdf.com/ https://upscpdf.com/
https://t.me/visioniastestseries2024
https://upscpdf.com/ https://upscpdf.com/

16

• To reduce the emissions intensity increasing living standards of the vast


of its GDP by 33 to 35 percent by majority of people of India and reducing
2030 from 2005 level. Hence, their vulnerability to the impacts of climate
option 1 is correct. change.
• To achieve about 40 per cent There are eight ‘National Missions’
cumulative electric power installed which form the core of the National
capacity from non-fossil fuel based Action Plan. They focus on promoting
energy resources by 2030, with the understanding of climate change,
help of transfer of technology and adaptation and mitigation, energy
low cost international finance, efficiency and natural resource
including from Green Climate conservation.
Fund. Hence, Option 3 is correct. 1. National Solar Mission
• To create an additional carbon sink 2. National Mission for Enhanced Energy
of 2.5 to 3 billion tonnes of CO2 Efficiency
equivalent through additional
3. National Mission on Sustainable
forest and tree cover by 2030.
Habitat
Hence, Option 2 is correct.
• To build capacities, create 4. National Water Mission
domestic framework and 5. National Mission for Sustaining the
international architecture for quick Himalayan Ecosystem
diffusion of cutting edge climate 6. National Mission for a Green India
technology in India and for joint 7. National Mission for Sustainable
collaborative R&D for such future Agriculture
technologies. 8. National Mission on Strategic
Knowledge for Climate Change
46. Ans: C Statement 2 is correct: The Principles of
Exp: the National Action Plan on Climate
Optimum Holding is the Maximum size of Change (NAPCC) are:
the holding which must be possessed and • Protecting the poor through an
owned by a family. inclusive and sustainable
Economic Holding is that holding which development strategy,
ensures a minimum satisfactory standard of sensitive to climate change
living in a family. In other words, economic • Achieving national growth and
holding is a minimum essential area for poverty alleviation objectives
profitable agriculture. Family holding while ensuring ecological
gives work to an average-sized family sustainability
having one plough under a traditional • Efficient and cost-effective
farming system. In other words, family strategies for end-use demand-side
holding is a ‘plough unit’ which is neither management
less nor more for an average size family to • Extensive and accelerated
cultivate properly. deployment of appropriate
technologies for adaptation and
47. Ans: B mitigation
Exp: • New and innovative market,
Statement 1 is incorrect: The National regulatory, and voluntary
Action Plan on Climate Change (NAPCC) mechanisms for sustainable
It outlines a national strategy that aims to development.
enable the country to adapt to climate
change and enhance the ecological 48. Ans: B
sustainability of India‘s development path. Exp:
It stresses that maintaining a high growth Statement 1 is incorrect: Integrated
rate is essential for Scheme for School Education (ISSE), i.e.
‘Samagra Shiksha’ was launched in

https://upscpdf.com/ https://upscpdf.com/
https://t.me/visioniastestseries2024
https://upscpdf.com/ https://upscpdf.com/

17

2018–19 by subsuming three erstwhile • Small ticket size coupled with high
Centrally Sponsored Schemes, namely transaction and service delivery
Sarva Shiksha Abhiyan (SSA), costs.
Rashtriya Madhyamik Shiksha Abhiyan • Absence of a business model that
(RMSA) and Teacher Education (TE). can attract good intermediaries.
The ISSE envisages school education as a • Capacity building of
continuum from preschool to senior intermediaries.
secondary level and aims to ensure • Lack of basic awareness and
inclusive and equitable quality education. knowledge on how insurance
works
Statement 2 is correct: Under it, States
and UTs are supported for strengthening of 50. Ans: C
existing government schools, and for the Exp:
creation and augmentation of infrastructure The Eighth Plan could not take off in 1990
facilities based on proposals received from due to the fast-changing political situation
respective States/UTs. The scheme at the Centre and the years 1990-91 and
includes an annual school grant to look 1991-92 were treated as Annual Plans. The
after a variety of needs of the school such Eighth Plan was finally launched in 1992
as—sanitation, annual maintenance, after the initiation of structural adjustment
toilets and other facilities to upkeep the policies.
infrastructure in good condition besides This was the first plan which went on for an
various interventions for improvement of introspection of the macro-economic
quality of education (such as induction and policies which the country had been
in-service training of teachers and school pursuing for many decades.
heads, learning enhancement programmes,
use of ICT in education, assessment of 51. Ans: B
learning outcomes, provision of libraries Exp:
and supplementary graded material to The Intergovernmental Panel on Climate
schools, etc). Change (IPCC) reviews and assesses the
most recent scientific, technical, and socio-
49. Ans: B economic information produced worldwide
Exp: relevant to climate change. The IPCC in its
Micro-insurance is specifically intended recent report the Fifth Assessment Report
for the protection of low-income people, (AR5), published in 2014, has observed
with affordable insurance products to help that there has been an increasing trend in
them cope with and recover from financial the anthropogenic emissions of greenhouse
losses. The need for insurance for an gases (GHG) since the advent of the
underprivileged section cannot be avoided industrial revolution, with about half of the
as this section of society is more prone to anthropogenic C02 emissions during this
many risks which ultimately leads to the period occurring in the last forty years.
incapacity to face such uncertain situations. The Fifth Assessment Report (AR5) from
Hence, the role that micro-insurance plays the Intergovernmental Panel on Climate
becomes inevitable. Change (IPCC) reveals the cumulative
The concept of microinsurance has been contributions to global CO2 emissions by
developed by the private insurance different countries. Contribution (%) by
company Aviva Life Insurance (in different countries to cumulative Global
partnership with MFIs), which has forged CO2: USA (21.2); EU (18.4); China
alliances with banks like Canara Bank, (10.7); Russia (7.4); Brazil (4.4); Japan
P&SB, RRBs, 23 cooperatives, etc., to (3.3); India (2.8); Canada (2.2); and Rest
promote microfinance. of the World (28.7).
The major challenges in microinsurance
distribution are: -

https://upscpdf.com/ https://upscpdf.com/
https://t.me/visioniastestseries2024
https://upscpdf.com/ https://upscpdf.com/

18

52. Ans: D 53. Ans: C


Exp: Exp:
The National Mission for Enhanced Energy The United States and the United
Efficiency (NMEEE) is one of the eight Kingdom announced the Atlantic
national missions under the National Declaration for a Twenty-First Century
Action Plan on Climate Change (NAPCC). U.S. - UK Economic Partnership to
NMEEE aims to strengthen the market for ensure that their unique alliance is
energy efficiency by creating conducive adapted, reinforced, and reimagined for
regulatory and policy regime and has the challenges of the time. U.S.-UK
envisaged fostering innovative and cooperation and joint leadership are as
sustainable business models to the energy essential as ever – both internationally and
efficiency sector. The Mission has been for the security and prosperity of the people
implemented since 2011. of both countries. The Atlantic Declaration
NMEEE consist of four initiatives to and accompanying Action Plan form the
enhance energy efficiency in energy basis of a new type of innovative
intensive industries which are as follows: partnership across the full spectrum of
• Perform Achieve and Trade economic, technological, commercial and
Scheme (PAT): Implementing a trade relations.
market assisted compliance Atlantic Declaration Action Plan
mechanism to accelerate (ADAPT): The plan encompasses five key
implementation of cost effective pillars as follows
improvements in energy efficiency 1. Leadership in critical and emerging
in large energy-intensive industries technologies.
• Market Transformation for 2. Cooperation on economic security
Energy Efficiency (MTEE): and technology protection toolkits
Accelerating the shift to energy and supply chains,
efficient appliances in specific 3. partnering on an inclusive and
application through innovative responsible digital transformation,
measures to make the products 4. building the clean energy economy
more affordable. of the future,
• Energy Efficiency Financing 5. further strengthening the alliance
Platform (EEFP): Facilitating across defence, health security, and
Financial Institutions to invest in space.
Energy Efficiency Projects and
Programmes 54. Ans: C
• Framework for Energy Efficient Exp:
Economic Development Statement 1 is correct: The United
(FEEED): Developing fiscal Nations Conference on Sustainable
instruments to leverage financing Development - or Rio+20 - took place in
for Energy Efficiency through risk Rio de Janeiro, Brazil in 2012. It resulted
mitigation: in a focused political outcome document
1. Partial Risk Guarantee Fund which contains clear and practical
for Energy Efficiency measures for implementing sustainable
(PRGFEE) and development.
2. Venture Capital Fund for In Rio, Member States decided to launch a
Energy Efficiency (VCFEE) to process to develop a set of Sustainable
promote energy efficiency Development Goals (SDGs), which will
3. National Energy Conservation build upon the Millennium Development
Award and Painting Goals and converge with the post 2015
Competition development agenda.
Statement 2 is correct: The Conference
adopted ground-breaking guidelines on
green economy policies. Governments also

https://upscpdf.com/ https://upscpdf.com/
https://t.me/visioniastestseries2024
https://upscpdf.com/ https://upscpdf.com/

19

decided to establish an intergovernmental enterprises became symbols of inefficiency


process under the General Assembly to and imposed a heavy burden on the
prepare options on a strategy for government through their perpetual losses.
sustainable development financing. The new industrial policy seeks to limit
Governments adopted the 10-year the role of the public sector and
framework of programmes on encourage the private sector’s
sustainable consumption and production participation in a wider field of
patterns, as contained in document industry.
A/CONF.216/5, and invited the General
Assembly, at its sixty-seventh session, to 56. Ans: B
designate a Member State body to take any Exp:
necessary steps to fully operationalize the Statement 1 is incorrect: Leptospirosis is
framework. a bacterial disease that affects humans
and animals. It is caused by bacteria of the
genus Leptospira. Without treatment,
55. Ans: C Leptospirosis can lead to kidney damage,
Exp: meningitis (inflammation of the membrane
In order to consolidate the gains already around the brain and spinal cord), liver
achieved during the 1980s and to provide failure, respiratory distress, and even death.
greater competitive stimulus to the Statement 2 is correct: The bacteria that
domestic industry, a series of reforms were cause leptospirosis are spread through
introduced in the Industrial Policy. The the urine of infected animals, which can
government announced a New Industrial get into water or soil and can survive
Policy on 24 July 1991. The New Industrial there for weeks to months. Many different
Policy established in 1991 sought kinds of wild and domestic animals carry
substantially to deregulate industry so as to the bacterium. These can include, but are
promote the growth of a more efficient and not limited to:
competitive industrial economy. Some • Cattle
elements of industrial policy reforms were: • Pigs
Statement 1 is correct: The new industrial • Horses
policy abolishes the system of industrial • Dogs
licensing for most of the industries under • Rodents
this policy no licences are required for When these animals are infected, they may
setting up new industrial units or for have no symptoms of the disease. Infected
substantial expansion in the capacity of the animals may continue to excrete the
existing units, except for a short list of bacteria into the environment continuously
industries relating to country’s security and or every once in a while for a few months
strategic concerns, hazardous industries up to several years.
and industries causing environmental Humans can become infected through:
degradation. • Contact with urine (or other body
Statement 2 is correct: The Industrial fluids, except saliva) from infected
Policy 1991 announced some more steps to animals.
promote rapid industrial development. It • Contact with water, soil, or food
said that the government would set up a contaminated with the urine of
special board (which was established as infected animals.
the Foreign Investments Promotion
Board—FIPB) to negotiate with a 57. Ans: A
number of international companies for Exp:
direct investment in industries in India. Statement 1 is incorrect: The Genetic
Statement 3 is correct: The public sector, Engineering Appraisal Committee
which was conceived as a vehicle for rapid (GEAC) functions in the Ministry of
industrial development, largely failed to do Environment, Forest and Climate
the job assigned to it. Most public sector Change (MoEF&CC). As per Rules,

https://upscpdf.com/ https://upscpdf.com/
https://t.me/visioniastestseries2024
https://upscpdf.com/ https://upscpdf.com/

20

1989, it is responsible for the appraisal of Statement 3 is correct: The ecological


activities involving large-scale use of impact of the Green Revolution was
hazardous microorganisms and profound. Soil fertility degradation, falling
recombinants in research and industrial water tables due to excessive irrigation
production from the environmental angle. requirements of new seed varieties, and
The committee is also responsible for the environmental degradation from the
appraisal of proposals relating to the release uncontrolled use of chemical inputs were
of genetically engineered (GE) organisms significant issues. This had long-term
and products into the environment, implications for the sustainability of
including experimental field trials. agriculture in the affected regions.
Statement 2 is incorrect: GEAC is
chaired by the Special 59. Ans: D
Secretary/Additional Secretary of Exp:
MoEF&CC and co-chaired by a Statement 1 is incorrect: The Bank for
representative from the Department of International Settlements (BIS) was
Biotechnology (DBT). Presently, it has 24 founded in 1930 as a clearinghouse for
members and meets every month to review German war reparations imposed by the
the applications in the areas concerned. Treaty of Versailles. It is an international
Statement 3 is correct: The functions of financial institution offering banking
GEAC as prescribed in the Rules 1989 are services for national central banks and a
as follows: forum for discussing monetary and
• To appraise activities involving regulatory policies. The BIS, which is
large-scale use of hazardous owned by 63 national central banks,
microorganisms and recombinants represents countries from around the world
in research and industrial that together account for about 95% of
production from the environmental world GDP. It also provides independent
angle. economic analysis.
• To appraise proposals relating to Statement 2 is incorrect: The original
the release of genetically members were Germany, Belgium,
engineered organisms and products France, Britain, Italy, Japan, the U.S.,
into the environment, including and Switzerland, and India was not a
experimental field trials. founding member of the bank.
• The committee or any persons Reparations were discontinued shortly after
authorized by it has the power to the bank's founding, and the BIS became a
take punitive action under the forum for cooperation and a counterparty
Environment Protection Act. for transactions among central banks.

58. Ans: A 60. Ans: A


Exp: Exp:
Statement 1 is incorrect: The Green The Extended Credit Facility (ECF)
Revolution, while boosting overall food provides medium-term financial assistance
production, exacerbated inter-personal and to low-income countries (LICs) with
inter-regional disparities in farmers' protracted balance of payments problems.
income in India. The benefits were not Statement 1 is correct: The purpose of the
uniformly distributed, leading to increased ECF is to assist PRGT-eligible countries
inequality among farmers. with a protracted balance of payments
Statement 2 is incorrect: The swing in problem to implement economic programs
cropping patterns towards wheat and rice that make significant progress toward a
during the Green Revolution marginalised stable and sustainable macroeconomic
the cultivation of pulses, oilseeds, maize, position consistent with strong and durable
and barley. This shift negatively impacted poverty reduction and growth. The ECF is
the production of these crops. one of the facilities under the Poverty

https://upscpdf.com/ https://upscpdf.com/
https://t.me/visioniastestseries2024
https://upscpdf.com/ https://upscpdf.com/

21

Reduction and Growth Trust (PRGT). It reducing the vulnerability of goods and
also aims to help catalyze foreign aid. persons to the effects of climate change.
All PRGT-eligible countries facing a Various areas in which Investments
protracted balance of payments problem made are considered as Green
(resolution of underlying macroeconomic Investment are:
imbalances is expected to extend over the • Waste Processing Recycling.
medium or longer term) can avail the • Climate Change Adaptation.
facility. • Biodiversity Protection.
Statement 2 is incorrect: • Water Sanitation
Conditionality of the Extended Credit • Industrial Pollution Control
Facility: • Renewable Energies
• Economic programs should
include policies to make progress 62. Ans: A
toward a stable and sustainable Exp:
macroeconomic position Pair 1 is correct: In India, brown bears are
consistent with strong and durable present in 23 protected areas (PA) in the
poverty reduction and growth. Union Territory of Jammu and Kashmir,
• Programs should be aligned with and the states of Himachal Pradesh and
the country’s own development Uttarakhand. Their global number is
strategy and aim to advance the estimated to be less than 1,000, and
country’s poverty reduction and possibly half of that in India. The
growth objectives. Himalayan brown bears are listed as
• Conditionality is focused on policy “critically endangered” in the
actions that are critical to achieving International Union for the
the program’s objectives. Conservation of Nature Red List.
• A Poverty Reduction and Growth Pair 2 is incorrect: Jerdon’s courser
Strategy (PRGS) should be issued Rhinoptilus bitorquatus is a Critically
to the IMF’s Executive Board for Endangered nocturnal cursorial bird
completion of the second and found only in Andhra Pradesh and
subsequent reviews. Telangana, India. It was considered to be
extinct from the beginning of the 20th
61. Ans: D century until its rediscovery in 1986. An
Exp: increase in anthropogenic activities has
Green finance is a broad term that can refer resulted in the loss of scrub jungle habitat
to financial investments flowing into at the fringes of the Sri Lankamalleswara
sustainable development projects and Wildlife Sanctuary, where the species is
initiatives, environmental products, and found.
policies that encourage the development of Pair 3 is incorrect: Pygmy possums are
a more sustainable economy. tree-dwelling marsupials. From head to tail,
Green finance includes climate finance but they can be as large as 12cm and, fully
is not limited to it. It also refers to a wider grown, as small as 5cm! They weigh
range of other environmental objectives, between 10g and 50g. They have large
for example industrial pollution control, eyes, large ears, and long whiskers. Their
water sanitation, or biodiversity soft, fur coat is fawn to grey on top and
protection. white underneath. Like many marsupials,
Mitigation and adaptation finance is their long tails swell with extra fat in times
specifically related to climate change of plenty. The Mountain Pygmy Possum
related activities: mitigation financial flows is considered Critically Endangered by
refer to investments in projects and the International Union for
programs that contribute to reducing or Conservation of Nature (IUCN).
avoiding greenhouse gas emissions
(GHGs) whereas adaptation financial flows
refer to investments that contribute to

https://upscpdf.com/ https://upscpdf.com/
https://t.me/visioniastestseries2024
https://upscpdf.com/ https://upscpdf.com/

22

• Assists exporters in recovering


63. Ans: D bad debts. Hence, statement 2 is
Exp: correct.
Statement 1 is correct: The multiplicity of • Provides information on the
cropping systems in Indian agriculture is a creditworthiness of overseas
distinctive feature, and it is attributed to buyers.
rainfed agriculture and the prevailing The Corporation has set before itself the
socio-economic situations of the farming following objectives:
community. 1. To encourage and facilitate the
Statement 2 is correct: The increased globalisation of India’s trade.
demand for food, resulting from population 2. To assist Indian exporters in
growth and urbanisation, puts agricultural managing their credit risks by
land under stress. In response, farmers providing timely information on
resort to crop intensification. the worthiness of the buyers,
Statement 3 is correct: Cropping systems bankers and the countries.
of a region are decided, by and large, by a 3. To protect the Indian exporters
number of soil and climatic parameters, against unforeseen losses, which
which determine the overall agro may arise due to failure of the
ecological setting for nourishment and buyer, bank or problems faced by
appropriateness of a crop or set of crops for the country of the buyer by
cultivation. providing cost-effective credit
insurance covers in the form of
Policy, Factoring and Investment
64. Ans: C Insurance Services comparable to
Exp: similar covers available to
ECGC Ltd. (Formerly Export Credit exporters in other countries.
Guarantee Corporation of India Ltd.), 4. To facilitate availability of
wholly owned by the Government of India, adequate bank finance to the
was set up in 1957 with the objective of Indian exporters by providing
promoting exports from the country by surety insurance covers for
providing Credit Risk Insurance and bankers at competitive rates.
related services for exports. It functions Hence, statement 3 is correct.
under the administrative control of the 5. To achieve improved performance
Ministry of Commerce & Industry and is in terms of profitability, financial
managed by a Board of Directors and operational efficiency
comprising representatives of the indicators and achieve optimum
Government, Reserve Bank of India, return on investment.
banking, and insurance and exporting 6. To develop world-class expertise
community. Hence, statement 1 is in credit insurance among
correct. employees ensure continuous
How does ECGC help exporters? innovation and achieve the highest
• ECGC Offers insurance protection customer satisfaction by delivering
to exporters against payment risks. top-quality service.
• Provides guidance in export- 7. To educate the customers through
related activities. continuous publicity and effective
• Makes available information on marketing.
different countries with its own
credit ratings. 65. Ans: D
• Makes it easy to obtain export Exp:
finance from banks/financial By the early 1990s, the need of the hour
institutions. was to change the nature and structure of
the economy. The Government of India
decided to change the very nature of the

https://upscpdf.com/ https://upscpdf.com/
https://t.me/visioniastestseries2024
https://upscpdf.com/ https://upscpdf.com/

23

industrial policy, which will automatically fertilisers, drugs, etc. Though a major
lead to a change in the nature and scope of section of the products under the APM
the economy. And then came the New were produced by the private sector, they
Industrial Policy of 1991. With this were not sold on market principles which
policy, the government kickstarted the very hindered the profitability of the
process of reform in the economy, which manufacturers as well as the sellers. It also
is why the policy is taken more as a hindered the expansion of the concerned
process than a policy. industries, leading to a demand-supply gap.
India was faced with a severe balance
of payment crisis by June 1991. Basically, Statement 2 is incorrect: State’s Role in
in the early 1990s, there were inter- the Reform: For the first time, an
connected set of events, which were important role of the state was designed,
growing unfavourable for the Indian in the process of economic reforms. All
economy: new steps of the reforms were now to be
• Due to the Gulf War (1990–91), started by the state, with the centre playing
the higher oil prices were fast a supportive role. It also favours greater
depleting India’s foreign reserves. fiscal leverage to the states.
• Sharp decline in the private
remittances from overseas Indian 67. Ans: B
workers in the wake of the Gulf Exp:
War, especially from the Gulf Pair 1 is correct: Turkmenistan
region. inaugurated Arkadag, a $ 5-billion
• Inflation peaking at nearly 17 "smart" city built in honour of former
per cent. strongman leader Gurbanguly
• The gross fiscal deficit of the Berdymukhamedov, cementing one of the
Central Government reached most extraordinary personality cults in the
around 8.4 per cent of the GDP. world. The new city built in the foothills of
• By the month of June 1991, India’s the picturesque Kopet-Dag mountains is
foreign exchange had declined to named after Gurbanguly
just two weeks of import Berdymukhamedov, who ruled over the
coverage. country between 2006 and 2022 and now
carries the title Hero Arkadag (Protector).
66. Ans: A Pair 2 is correct: Kaas Plateau
Exp: (Maharashtra, India) – also known as
The government launched the second “Maharashtra’s Valley of Flowers” – is a
generation of the reforms in the year 2000- 365-day tourist attraction due to its
01. Basically, the reforms India launched in scenic beauty. The Karvi flowers bloom
the early 1990s were not taking place as only once in every seven or eight years. The
desired, and a need for another set of Karvi are also found in Karnataka and
reforms was felt by the government which Kerala however, the flowering period in
were initiated with the title of the Second these states is one in every 10 or 12 years.
Generation of economic reforms. This By contrast, these flowers bloom once
generation's reforms were deeper and more every seven or eight years in Maharashtra
delicate and required higher political due to the salinity, hot and humid weather,
willpower from the governments. and height above sea level.
Statement 1 is correct: Considered as the Pair 3 is incorrect: Khat-ar-shnong
‘backbone’ for the success of the reform refers to a valley region in East Khasi
process in India itself, it consists of the Hills, Meghalaya (India) which has been
dismantling of the Administered Price isolated for a long time due to the terrain.
Mechanism (APM). Kongthong is a prominent village because
There were many products in the economy of its striking location and its unique
whose prices were fixed/regulated by the tradition where residents of the village are
government, viz., petroleum, sugar, assigned a tune that also serves as a unique

https://upscpdf.com/ https://upscpdf.com/
https://t.me/visioniastestseries2024
https://upscpdf.com/ https://upscpdf.com/

24

ID. The residents of the Kongthong, known was introduced in Lok Sabha on August 4,
as the Whistling Village, along with people 2023. It repeals the Science and
from the nearby villages, have been Engineering Research Board Act, 2008,
practising this tradition which is called the and dissolves the Science and
‘jingrwai iawbei’ for generations. Engineering Research Board set up
under it. The Bill provides for establishing
68. Ans: D the Anusandhan National Research
Exp: Foundation (NRF). NRF will be the apex
Disinvestment: It is the process of ‘selling body in the country to provide strategic
ownership’ in a company. Technically, direction for research, innovation, and
the term may be used in the case of any entrepreneurship in the fields of
company (i.e., privately owned company), (i) natural sciences, including
but in practice, it is used only in the case of mathematics,
a government-owned company. (ii) engineering and technology,
Disinvestment commenced in the country
(iii) environmental and earth sciences,
with three interrelated co-ordinates:
• As a tool of public sector (iv) health and agriculture, and
reforms; (v) scientific and technological interfaces
• As a part of the economic reform of humanities and social sciences.
process (i.e., as part of the de- Statement 2 is incorrect: The Foundation
reservation of industries) will be financed through:
• As a tool of resource mobilisation • Grants and loans from the
for budgetary needs. central government
The Department of Investment and Public • Donations to the fund
Asset Management (DIPAM), which • Income from investments of the
comes under the Finance Ministry, defines amounts received by the
Strategic disinvestment as follows: Foundation
“Strategic disinvestment would imply • All amounts with the Fund for
the sale of a substantial portion of the Science and Engineering
Government shareholding of a central Research set up under the 2008
public sector enterprise (CPSE) of up to Act.
50%, or such higher percentage as the Statement 3 is correct: NRF will have a
competent authority may determine, along Governing Board headed by the Prime
with transfer of management control. Minister of India. The Board will provide
• Strategic Disinvestment is to be strategic direction to the Foundation and
done through consultation among monitor the implementation. Other
Ministries/Departments and members of the Board are:
NITI Aayog. • The Union Ministers of Science
• NITI Aayog to identify PSUs and and Technology, Education as
advise on their different aspects. Vice Presidents
• CGD (Core Group of Secretaries • The Principal Scientific Advisor as
on Disinvestment) to consider the Member Secretary
recommendations of NITI Aayog • Secretaries to the Departments of
to facilitate a decision by the Science and Technology,
CCEA (Cabinet Committee on Biotechnology, and Scientific and
Economic Affairs) and to Industrial Research.
supervise/monitor the
implementation process. 70. Ans: C
Exp:
69. Ans: B Pradhan Mantri Kaushal Vikas Yojana
Exp: (PMKVY) was launched in 2015 to
Statement 1 is correct: The Anusandhan encourage and promote skill development
National Research Foundation Bill, 2023 in the country by providing free short-
duration skill training and incentivising

https://upscpdf.com/ https://upscpdf.com/
https://t.me/visioniastestseries2024
https://upscpdf.com/ https://upscpdf.com/

25

this by providing monetary rewards to (QPs)/National Occupational


youth for skill certification. It is the Standards (NOSs).
flagship scheme of the Ministry of Skill
Development and Entrepreneurship 71. Ans: C
(MSDE). Exp:
Objectives of PMKVY 2016-20: Statement 1 is correct: Digital Publisher
• Enable and mobilise a large Content Grievances Council (DPCGC) is
number of youths to take up an officially recognized, independent self-
industry-designed quality skill regulatory body for online curated content
training, become employable and providers (OCCPs). It was set up under
earn their livelihood. the aegis of the Internet and Mobile
• Increase productivity of the Association of India (IAMAI), in June
existing workforce, and align skill 2021. DPCGC, through the Grievance
training with the actual needs of the Redressal Board (GRB), DPCGC aims to
country. usher in a redressal mechanism which
• Encourage standardisation of the ensures a balance between addressing
Certification process and put in viewer complaints fairly and showcasing
place the foundation for creating a content without ad-hoc interventions.
registry of skills. Statement 2 is correct: DPCGC is
• Benefit 10 million youth over the registered by the Ministry of
period of four years (2016- 2020). Information & Broadcasting as a Level-
Key components of the Scheme: II Self-Regulatory Body for publishers of
• Short-Term Training: The Short- online curated content, under the
Term Training imparted at Information Technology (Intermediary
PMKVY Training Centres (TCs) is Guidelines and Digital Media Ethics
aimed towards the candidates who Code) Rules, 2021. It adheres to IT Rules
are either school/college dropouts 2021 and other existing or new statutes, and
or unemployed. The Training is rules/regulations/guidelines framed
provided according to the National thereunder from time to time, relating to
Skills Qualification Framework publishing of online curated content.
(NSQF) with Soft Skills,
Entrepreneurship, Financial and 72. Ans: A
Digital Literacy curriculum, a part Exp:
of the curriculum. Upon Statement 1 is correct: Disinvestment is
successful completion of their the process of ‘selling ownership’ in a
assessment and certification, company. In January 2005, the
candidates are provided Government of India decided to constitute
placement assistance by Training a ‘National Investment Fund’ (NIF) into
Partners (TPs). which the proceeds from the disinvestment
• Recognition of Prior Learning of Central Public Sector Enterprises were to
(RPL) - Individuals with prior be channelized.
learning experience or skills are Statement 2 is incorrect:
assessed and certified under the The salient features of NIF are:
Recognition of Prior Learning • The proceeds from
(RPL) component of the Scheme. disinvestment will be
• Special Projects: Special Projects channelized into the NIF, which
component of PMKVY envisages is to be maintained outside the
encouraging training in special Consolidated Fund of India.
areas and premises of Government • The corpus of the National
bodies, corporates/industry bodies Investment Fund will be of a
and training in special job roles not permanent nature.
defined under the available • The Fund will be professionally
Qualification Packs managed, to provide sustainable

https://upscpdf.com/ https://upscpdf.com/
https://t.me/visioniastestseries2024
https://upscpdf.com/ https://upscpdf.com/

26

returns without depleting the on behalf of Interpol Member countries.


corpus, by selected Public sector The C.B.I. is not a statutory body. Its
Mutual Funds. investigative and jurisdiction powers are
• 75 percent of the annual income of governed by the DSPE Act, 1946.
the Fund will be used to finance Statement 2 is correct: It must
selected social sector schemes, mandatorily obtain the consent of the
which promote education, health state government concerned before
and employment. The residual 25 beginning to investigate a crime in a
per cent of the annual income of state. Section 6 of The DSPE Act
the Fund will be used to meet the (“Consent of State Government to exercise
capital investment requirements of of powers and jurisdiction”) says: “Nothing
profitable and revivable PSUs that contained in section 5 (titled “Extension of
yield adequate returns, in order to powers and jurisdiction of special police
enlarge their capital base to finance establishment to other areas”) shall be
expansion/diversification. deemed to enable any member of the Delhi
Special Police Establishment to exercise
73. Ans: C powers and jurisdiction in any area in a
Exp: State, not being a Union territory or railway
Globalisation intends to integrate the area, without the consent of the
Indian Economy with the world economy. Government of that State.”
The four parameters of globalisation are (i) Statement 3 is correct: The first state to
trade flows. (ii) capital flows, (iii) withdraw consent was Mizoram in 2015.
technology flows, and (iv) labour flows. Eight other states had withdrawn consent to
The measures undertaken for the the CBI: Maharashtra, Punjab, Rajasthan,
promotion of globalisation in India West Bengal, Jharkhand, Chhattisgarh,
include Kerala, and Mizoram. The CBI’s position
(i) Reduction of import duties is, in this respect, different from that of the
(ii) Encouragement of foreign National Investigation Agency (NIA),
investments, and granting of which is governed by The NIA Act, 2008,
automatic approval for direct foreign and has jurisdiction across the country.
investment in various sectors.
75. Ans: D
(iii)Encouragement of foreign
Exp:
technology agreements
As per the MSME Act, 2006 the MSMEs
Delicensing was an important
are classified into two classes:
component of Economic Reforms. The
Manufacturing and service enterprises—
number of items requiring licensing was
and they are defined in terms of investment
reduced to a short list of bare 4 industries.
in plant & machinery. The Micro, Small
This freed the private sector to set up
and Medium Enterprises (MSMEs) play a
industrial units quickly.
very vital role in the economy.
The sector has huge potential to help
74. Ans: A
address structural problems like
Exp:
unemployment, regional imbalances, and
Statement 1 is incorrect: The Central
unequal distribution of national income and
Bureau of Investigation (CBI), functioning
wealth. Due to comparatively low capital
under Dept. of Personnel, Ministry of
costs and their forward-backwards linkages
Personnel, Pension & Public Grievances,
with other sectors, they are headed to play
Government of India, is the premier
a crucial role in the success of the Make in
investigating police agency in India. It is an
India initiative.
elite force playing a major role in the
Enhancing ease of doing business for the
preservation of values in public life and in
sector and their faster growth have been
ensuring the health of the national
given special attention by the Government
economy. It is also the nodal police agency
in India, which coordinates investigations

https://upscpdf.com/ https://upscpdf.com/
https://t.me/visioniastestseries2024
https://upscpdf.com/ https://upscpdf.com/

27

in the last few years with the following to enrich the religious tourism
policy steps taken till early 2020: experience. It aims to integrate pilgrimage
• Loans up to ₹1 crore to be destinations in a prioritised, planned and
approved online within 59 sustainable manner to provide a complete
minutes; religious tourism experience. The growth
• Interest subvention of 2 per cent to of domestic tourism hugely depends on
all GST registered firms; pilgrimage tourism.
• PSUs to compulsorily procure 25
per cent (up from 20 per cent) with
3 per cent reserved for women 77. Ans: C
entrepreneurs—through GeM Exp:
portal; It was in the year 2000-01 that the
• 20 Technology Centres (TCs) and government, for the first time, announced
100 Extension Centres (ECs) to be the need for the Second Generation of
established; economic reforms and it was launched the
The share of MSME manufacturing same year. The ones which had been
output in all India Manufacturing initiated by then (i.e., from 1991 to 2000)
output during the year 2019-20, 2020-21 were called by the government as the
and 2021-22 was 36.6%, 36.9% and reforms of the First Generation.
36.2% respectively and can be seen as The External Sector Reforms under the
fluctuating. second generation economic reforms
The share of MSME Gross Value Added consisted of policies like—
(GVA) in India's Gross Domestic Product • Abolishing quantitative restrictions
(GDP) during the years 2019-20, 2020-21, on import
and 2021-22 was 30.5%, 27.2% and 29.2%, • switching to the floating currency
respectively. regime of exchange rate
• announcing full current account
76. Ans: D convertibility (not capital account
Exp: convertibility)
Statement 1 is correct: In light of India's • reforms in the capital account
G20 Presidency and the India@75 Azadi ka • permission to foreign investment
Amrit Mahotsav celebrations, the Ministry (direct as well as indirect),
of Tourism has designated 2023 as the • promulgation of a liberal Foreign
'Visit India Year' to promote inbound Exchange Management Act (the
travel. FEMA replacing the FERA)
Statement 2 is correct: India allows
100% Foreign Direct Investment (FDI) 78. Ans: A
in the tourism industry under the Exp:
automatic route. Additionally, 100% FDI Statement 1 is correct: This is a very
is permitted for tourism construction flexible method of giving contracts (i.e.,
projects, including the development of public procurement) which can be used in
exquisite hotels, resorts, and unparalleled PPP as well as non-PPP projects.
recreational facilities. Statement 2 is incorrect: In this, one
Statement 3 is correct: bidder is asked by the government to
PRASAD Scheme: submit the proposal for the project, which
The Government of India launched the is put in the public domain. Afterwards,
PRASAD scheme in the year 2014-2015 several other bidders submit their
under the Ministry of Tourism. The full proposals aimed at improving and beating
form of the PRASAD scheme is the original (first) bidder—finally, an
‘Pilgrimage Rejuvenation And Spiritual improved bid is selected (called a counter-
Augmentation Drive’. proposal). If the original bidder is not able
This scheme focuses on developing and to match the counter-proposal, the project
identifying pilgrimage sites across India is awarded to the counter-bidder.

https://upscpdf.com/ https://upscpdf.com/
https://t.me/visioniastestseries2024
https://upscpdf.com/ https://upscpdf.com/

28

Statement 3 is incorrect: The Government 80. Ans: B


of India, for the first time, announced the Exp:
use of this model for redevelopment of Statement 1 is incorrect: India has the
railway stations in the country (by late 4th largest railway system in the world,
2015). Though the Government of India behind only the US, Russia and China. The
used this model for the first time, this has Indian Railways consists of a total track
already been used by several states by length of 126,366 km with 7,335 stations.
now—Karnataka, Andhra Pradesh, Statement 2 is correct:
Rajasthan, Madhya Pradesh, Bihar, Punjab The key objectives of the National Rail
and Gujarat—for roads and housing Plan are:-
projects. • Formulate strategies based on
both operational capacities and
79. Ans: B commercial policy initiatives to
Exp: increase the modal share of the
The investment climate in India has Railways in freight to 45%.
improved considerably since the opening • Reduce transit time of freight
up of the economy in 1991. This is substantially by increasing the
primarily attributed to the ease of FDI average speed of freight trains to
rules in India. India, today is a part of the 50 Kmph.
top 100 clubs on Ease of Doing Business • Identify new Dedicated Freight
(EoDB). Corridors.
Mauritius (24%), Singapore (23%), USA • Identify new High Speed Rail
(9%), Netherlands (7%) and Japan (6%) Corridors.
emerge as the top 5 countries for FDI • Assess rolling stock requirement
equity inflows into India in FY 2022-23. for passenger traffic as well as
The top 5 sectors receiving the highest FDI wagon requirement for freight.
Equity Inflow during FY 2022-23 are • Assess Locomotive requirements
Services Sector (Finance, Banking, to meet twin objectives of 100%
Insurance, Non-Fin/ Business, electrification (Green Energy) and
Outsourcing, R&D, Courier, Tech. Testing increasing freight modal share etc.
and Analysis, Other) (16%), Computer Statement 3 is correct: Till 22nd August
Software & Hardware (15%), Trading 2014, Foreign Direct Investment (FDI) in
(6%), Telecommunications (6%) and the Railway Sector was restricted only to
Automobile Industry (5%). the “Mass Rapid Transport System”.
Automatic Route: Under the Automatic However, FDI has now been opened
Route, the non-resident investor or the up/permitted in the following
Indian company does not require any activities/areas of Railway Infrastructure
approval from the Government of India for (100% on automatic route) under the
the investment. extant FDI Policy of Government since 22
FDI is prohibited in certain sectors like the August 2014:-
Lottery Business, including “Construction, Operation and
Government/private lottery, online Maintenance of:-
lotteries, Gambling and Betting, Chit i. Suburban corridors project through
Funds, Nidhi Companies, Trading in PPP.
Transferable Development Rights(TDR), ii. High-Speed Train Projects.
Real Estate Business or Construction of iii. Dedicated Freight Lines.
farmhouses, etc. iv. Rolling Stock, including train sets
and locomotives/coaches
manufacturing and maintenance
facilities.
v. Railway Electrification.
vi. Signalling systems.
vii. Freight terminals.

https://upscpdf.com/ https://upscpdf.com/
https://t.me/visioniastestseries2024
https://upscpdf.com/ https://upscpdf.com/

29

viii. Passenger terminals. Paris, France, on 12 December 2015. It


ix. Infrastructure in industrial park entered into force on 4 November 2016.
pertaining to railway line/ sidings • The Paris Agreement reaffirms
including electrified railway lines that developed countries should
and connectivities to main railway take the lead in providing
line. financial assistance to countries
x. Mass Rapid Transport Systems. that are less endowed and more
vulnerable, while for the first time
also encouraging voluntary
81. Ans: D contributions by other Parties.
Exp: Statement 1 is correct.
Statement 1 is correct: Disinvestment is • Climate finance is needed for
a process of selling government equity in mitigation, because large-scale
public sector enterprises. Disinvestment investments are required to
in India is seen as connected to three major significantly reduce emissions.
interrelated areas, namely Climate finance is equally
• A tool of public sector reforms important for adaptation, as
• A part of the economic reforms started significant financial resources are
in mid-1991. It has to be done as a needed to adapt to the adverse
complementary part of the ‘de- effects and reduce the impacts of a
reservation of industries’. changing climate.
• Initially motivated by the need to raise • The Paris Agreement speaks of the
resources for budgetary allocations. vision of fully realising technology
Statement 2 is correct: The term development and transfer for both
Privatisation in a very wide sense means improving resilience to climate
all the economic policies which directly change and reducing GHG
or indirectly seem to promote the emissions. It establishes a
expansion of the private sector or the technology framework to provide
market (economy) have been termed by overarching guidance to the well-
experts and governments as the process of functioning Technology
privatisation. We may cite a few examples Mechanism. The mechanism is
from India: de-licencing and de-reservation accelerating technology
of the industries even cuts in the subsidies development and transfer through
and permission to foreign investment. its policy and implementation
The process of Privatisation includes arms.
selling the shares of the state-owned • Not all developing countries have
enterprises to the private sector. sufficient capacities to deal with
If an asset has been sold out by the many of the challenges brought by
government to the tune of only 49 per cent climate change. As a result, the
the ownership remains with the state Paris Agreement places great
though it is considered privatisation. If the emphasis on climate-related
sale of shares of the state-owned assets has capacity-building for developing
been to the tune of 51 per cent, the countries and requests all
ownership is really transferred to the developed countries to enhance
private sector even then it is termed as support for capacity-building
privatisation. actions in developing countries.
Statement 2 is incorrect.
82. Ans: A
Exp:
The Paris Agreement is a legally binding
international treaty on climate change. It
was adopted by 196 Parties at the UN
Climate Change Conference (COP21) in

https://upscpdf.com/ https://upscpdf.com/
https://t.me/visioniastestseries2024
https://upscpdf.com/ https://upscpdf.com/

30

83. Ans: D dark energy, has passed its preliminary


Exp: design review. This clears the way for
Make in India was launched in September construction to begin. Euclid is designed to
2014 by the Government of India to give us important new insights into the
encourage multinational as well as "dark side" of the universe -- namely dark
domestic companies to manufacture their matter and dark energy, both thought to be
products in India. The initiative is set to key components of our cosmos.
boost entrepreneurship, not only in Statement 2 is correct: Euclid is a
manufacturing but in relevant European Space Agency mission with
infrastructure and service sectors as well. important contributions from NASA,
The initiative is based on four pillars: including infrared detectors for one
• New process instrument and science and data
• New infrastructure analysis. The Euclid consortium will
• New sectors provide science instruments and data and
• New mindset science analysis. NASA's Euclid Project
The initiative also aims at imposing high- Office is based at JPL (Jet Propulsion
quality standards and the dimensions of Laboratory). JPL will provide the infrared
sustainability. Key policies to be followed flight detectors for one of Euclid's two
are ease of doing business, getting away science instruments.
with archaic laws, 100 Smart Cities,
disinvestment of the PSUs, skills and jobs 86. Ans: C
for the youth, etc. Exp:
Statement 1 is correct: The term
Major challenges to the initiatives unorganised worker has been defined under
include: the Unorganized Workers' Social
• creating a healthy business Security Act, 2008, as a home-based
environment, worker, self-employed worker or a wage
• removal of unfavourable factors, worker in the unorganised sector and
• more focus on India’s MSMEs, includes a worker in the organised sector
• lack of world-class research and who is not covered by any of these Acts:
development (R&D), the Employee's Compensation Act, 1923,
• comparisons with China’s ‘Made the Industrial Disputes Act, 1947, the
in China’ campaign Employees' State Insurance Act, 1948 , the
Employees Provident Funds and
84. Ans: C Miscellaneous Provision Act, 1952 , the
Exp: Maternity Benefit Act, 1961 and the
The Forex Reserves (short for ‘foreign Payment of Gratuity Act, 1972.
exchange reserves’) of an economy are Statement 2 is correct: The unorganised
its ‘foreign currency assets’ added with workers suffer from cycles of excessive
its gold reserves, SDRs (Special Drawing seasonality of employment, lack of formal
Rights) and Reserve Tranche in the IMF. employer-employee relationship and
In a sense, the Forex reserves are the upper absence of social security protection.
limit up to which an economy can manage Several legislations, such as the
foreign currency in normal times if need be. Employee's Compensation Act, 1923; the
As per an RBI release of Nov 2023, India’s Minimum Wages Act, 1948; the Maternity
forex reserves were at around US $ 590 BenefitAct, 1961; the Contract Labour
Billion. {Abolition and Prohibition) Act, 1970; the
Building and Other Construction Workers
85. Ans: C (Regulation of Employment & Conditions
Exp: of Service) Act, 1996; and the Building and
Statement 1 is correct: Euclid, a planned Other Construction Workers Welfare
mission to investigate the profound (Cess)Act, 1996 etc.
cosmic mysteries of dark matter and

https://upscpdf.com/ https://upscpdf.com/
https://t.me/visioniastestseries2024
https://upscpdf.com/ https://upscpdf.com/

31

87. Ans: A
Exp: 88. Ans: A
Stand up India Scheme was launched on 5th Exp:
April 2016 to promote entrepreneurship at The UDISE+ system of online data
the grassroots level, focusing on economic collection from the schools was developed
empowerment and job creation. This by the Department of School Education
scheme has been extended up to the year & Literacy in the year 2018-19 to
2025. overcome the issues related to the erstwhile
Stand-Up India was launched to practice of manual data filling in paper
promote entrepreneurship amongst format. In UDISE+ system, improvements
women, Scheduled Castes (SC) & have been made particularly in the areas
Scheduled Tribes (ST) categories, to help related to data capture, data mapping and
them in starting a greenfield enterprise in data verification. In UDISE+ 2021-22,
manufacturing, services or the trading additional data on important indicators viz.,
sector and activities allied to agriculture. digital library, peer learning, hard spot
Hence, statement 1 is correct. identification, number of books available in
The scheme aims to encourage all bank the school library, etc have been collected
branches to extend loans to borrowers from for the first time to align with the NEP 2020
SC, ST and women in setting up their own initiatives.
greenfield enterprises.
Entities eligible for a loan: 89. Ans: B
• SC/ST and/or women Exp:
entrepreneurs, above 18 years of Statement 1 is incorrect: The RBI
age; officially targets neither a particular
• Loans under the scheme are exchange rate nor foreign exchange
available for only green field reserves. Its interventions aim to reduce
projects. volatility in the forex market rather than
(Greenfield signifies, in this maintaining a specific exchange rate.
context, the first-time venture of Statement 2 is correct: When RBI
the beneficiary in manufacturing, purchases dollars in the spot market to
services or the trading sector and support the weakening rupee, it leads to an
activities allied to agriculture) infusion of rupees into the system,
Hence, statement 2 is incorrect. potentially causing inflationary effects. To
mitigate this, RBI converts spot purchases
into forwards, incurring direct costs due to
• In the case of non-individual forward premiums. If RBI opts for open
enterprises, 51% of the market operations (OMOs) to mop up
shareholding and controlling excess liquidity, that also involves costs.
stake should be held by either
SC/ST and/or Women 90. Ans: B
Entrepreneur. Hence, statement 3 Exp:
is incorrect. Revamped Distribution Sector Scheme
• Borrowers should not be in default (RDSS) was launched in July 2021 with the
to any bank/financial institution; objective of improving the quality and
• The Scheme envisages ‘up to 15%’ reliability of power supply to consumers
margin money, which can be through a financially sustainable and
provided in convergence with operationally efficient distribution
eligible Central/State schemes. In Sector. Hence, statement 1 is incorrect.
any case, the borrower shall be
required to bring in a minimum The salient features of the scheme are
of 10 % of the project cost as i. The scheme aims to reduce the
their own contribution. AT&C losses to pan-India levels

https://upscpdf.com/ https://upscpdf.com/
https://t.me/visioniastestseries2024
https://upscpdf.com/ https://upscpdf.com/

32

of 12-15% and the ACS-ARR and foreign currencies in the concerned


(Average Cost of Supply - Average economy.
Realizable Revenue) gap to zero by Statement 3 is correct: A managed-
2024-25. exchange-rate system is a hybrid or mixture
ii. The Scheme has two major of the fixed and flexible exchange rate
components: Part ‘A’ – Financial systems in which the government of the
support for Prepaid Smart economy attempts to affect the exchange
Metering and System Metering and rate directly by buying or selling foreign
upgradation of the Distribution currencies or indirectly, through monetary
Infrastructure, and Part ‘B’ – policy (i.e., by lowering or raising interest
Training and Capacity Building rates on foreign currency bank accounts,
and other Enabling and supporting affecting foreign investment, etc.). Today,
Activities. most of the economies have shifted to this
iii. The release of funds under the system of exchange rate determination.
scheme has been linked to Almost all countries tend to intervene when
Results and Reforms. The pre- the markets become disorderly, or the
qualifying criteria need to be fundamentals of economics are challenged
mandatorily met by the DISCOMs by the exchange rate of the time.
before they can be evaluated for
release of funds under the scheme. 92. Ans: B
Hence, statement 2 is correct. Exp:
iv. The scheme has a duration of 5 Statements 1 is incorrect: The Ministry
years (FY 2021-22 to FY 2025-26). of Power and the Ministry of New and
The sunset date of the scheme Renewable Energy have jointly launched
will be 31.03.2026. Hence, a National Mission to quickly identify
statement 3 is incorrect. emerging technologies in the power
REC Limited and Power Finance sector and develop them indigenously, at
Corporation Limited (PFC) have scale, for deployment within and outside
been appointed as the Nodal Agencies India. The National Mission, titled
for the scheme and have been made “Mission on Advanced and High-Impact
responsible for facilitating the Research (MAHIR)”, aims to facilitate
implementation of the scheme in the indigenous research, development, and
entire country. demonstration of the latest and emerging
technologies in the power sector.
91. Ans: D Statement 2 is correct: The key objectives
Exp: of the Mission are as follows:
Statement 1 is correct: A fixed exchange • To identify emerging
rate is a regime applied by a government or technologies and areas of future
central bank that ties the country's official relevance for the Global Power
currency exchange rate to another country's Sector and take up indigenous
currency or the price of gold. The purpose end-to-end development of
of a fixed exchange rate system is to keep relevant technologies.
a currency's value within a narrow band. • To provide a common platform for
Statement 2 is correct: In the floating Power Sector Stakeholders for
exchange rate system, a domestic collective brainstorming, and
currency is left free to float against a synergetic technology
number of foreign currencies in its development and devise pathways
foreign exchange market and determine for smooth transfer of technology
its own value. Such exchange rates are also • To support pilot projects of
called market-driven or based exchange indigenous technologies
rates, which are regulated by factors such (developed especially by Indian
as the demand and supply of the domestic Start-ups) and facilitate their
commercialisation.

https://upscpdf.com/ https://upscpdf.com/
https://t.me/visioniastestseries2024
https://upscpdf.com/ https://upscpdf.com/

33

• To seed, nurture and scale up as part of the U.S.-India initiative on


scientific and industrial R&D and Critical and Emerging Technology (iCET).
to create a vibrant & innovative India’s Innovations for Defense Excellence
ecosystem in the Power Sector of (iDEX) and the Office of the Secretary of
the country. Defense (OSD) are leading INDUS-X
Statement 3 is correct: The Mission will activities for MoD and DoD, respectively.
have a two-tier structure - a Technical
Scoping Committee and an Apex 94. Ans: B
Committee. Exp:
• The Technical Scoping In the 1992-93 financial year, India
Committee, chaired by the moved to the floating currency regime
Chairperson of Central with its own method which is known as
Electricity Authority, will the ‘dual exchange rate’. There are two
identify ongoing and emerging exchange rates for the rupee, one is the
research areas globally, ‘official rate’, and the other is the ‘market
recommend potential technologies rate’. It should be noted that it is the every
for development under the day’s changing market-based exchange
Mission, justify the techno- rate of the rupee which affects the official
economic advantages, provide exchange rate and not the other way
research outlines, and conduct around. However, the RBI may intervene in
periodic monitoring of approved the forex market via the demand and supply
research projects. of the rupee or foreign currencies. None of
• The Apex Committee, chaired by the economies have to date followed an
the Union Minister for Power & ideal free-floating exchange rate
New and Renewable Energy, will
deliberate on the technology and 95. Ans: D
products to be developed and Exp:
approve the research proposals. Under the process of economic reforms, an
The Apex committee will also look Insurance Reforms Committee (IRC)
into international collaborations. was set up in April 1993 under the
chairmanship of the ex RBI Governor R.
93. Ans: B N. Malhotra. The committee handed over
Exp: its report (January 1994) with the following
Statement 1 is incorrect: The U.S. major suggestions:
Department of Defense (DoD) and the • Decontrolling the insurance sector,
Indian Ministry of Defense (MoD) i.e., allowing Indian as well as
launched the India-U.S. Defense foreign private sector insurance
Acceleration Ecosystem (INDUS-X). companies to enter the sector (the
Indian and U.S. defence firms intend to government did it in 1999 passing
identify opportunities to establish formal the IRDA Act).
and informal mentoring with start-ups to • Restructuring the LIC and the GIC
assist with market access, business and cutting down the government’s
strategy, and technology know-how. holding in them to 50 per cent (no
Statement 2 is correct: The initiative follow-up still, but the private
aims to expand the strategic technology insurance companies demanding it
partnership and defence anxiously).
industrial cooperation between • Delinking GIC and its four
governments, businesses, and academic subsidiaries (which was done in
institutions. This initiative builds on a 2000).
commitment by the U.S. and Indian • Discarding the system of licensing
National Security Advisors in January 2023 of surveyors by the controller of
to launch an “Innovation Bridge” to Insurance.
connect U.S. and Indian defence start-ups

https://upscpdf.com/ https://upscpdf.com/
https://t.me/visioniastestseries2024
https://upscpdf.com/ https://upscpdf.com/

34

• Restructuring the Tariff Advisory 1. Mobile Manufacturing and


Committee. Specified Electronic Components
2. Critical Key Starting
96. Ans: B Materials/Drug Intermediaries &
Exp: Active Pharmaceutical Ingredients
Statement 1 is correct: The external 3. Manufacturing of Medical Devices
sector's current account, maintained by 4. Automobiles and Auto
the central banking body on behalf of the Components
government, encompasses various 5. Pharmaceuticals/Drugs
transactions such as export, import, 6. Specialty Steel
interest payments, private remittances, 7. Telecom & Networking Products
and transfers. These transactions are 8. Electronic/Technology Products
recorded as either inflow or outflow (credit 9. White Goods (ACs and LEDs)
or debit), and the resulting balance can be 10. Food Products
positive (surplus) or negative (deficit) at the 11. Textile Products: MMF segment
end of the year. and technical textiles
Statement 2 is incorrect: India had a 12. High-efficiency solar PV modules
surplus current account for three 13. Advanced Chemistry Cell (ACC)
consecutive years (2000—03), not a Battery
deficit. This period was unique in Indian 14. Drones and Drone Components.
economic history as the country
experienced a positive balance in its current 98. Ans: B
account. Exp:
Statement 3 is incorrect: Current account The capital account, in international
deficit is shown either numerically by macroeconomics, is part of the balance of
showing the total monetary amount of the payments that records all transactions made
deficit, or in percentage of the GDP of the between entities in one country with
economy for the concerned year. Both the entities in the rest of the world. These
data are used in the analysis as per the transactions consist of imports and exports
specific requirement. As per an RBI of goods, services, capital, and transfer
release of April 2014, the sustainable payments such as foreign aid and
level of the current account deficit for remittances.
India is 2.5 percent of the GDP. Statement 1 is incorrect: The capital
account does not record the balance of trade
97. Ans: D and services. The current account captures
Exp: the balance of trade and services.
The purpose of the PLI Schemes is to Statement 2 is correct: Every transaction
attract investments in key sectors and in foreign currency (inflow or outflow)
cutting-edge technology; ensure efficiency considered as capital is shown in this
and bring economies of size and scale in the account - external lending and borrowing,
manufacturing sector and make Indian foreign currency deposits of banks, external
companies and manufacturers globally bonds issued by the Government of India,
competitive. Foreign Direct Investment (FDI), Portfolio
The PLI scheme is expected to have a Investment Scheme (PIS) and security
cascading effect on the country's MSME market investment of the Qualified Foreign
ecosystem. The anchor units that will be Investors (QFIs).
built in every sector are likely to set a new
supplier/vendor base in the entire value 99. Ans: C
chain. Most of these ancillary units are Exp:
expected to be built in the MSME sector. Statement 1 is correct: The Commission
14 sectors for which PLI schemes have of Railway Safety Working Under the
been approved are: Ministry of Civil Aviation of the
Government of India, deals with matters

https://upscpdf.com/ https://upscpdf.com/
https://t.me/visioniastestseries2024
https://upscpdf.com/ https://upscpdf.com/

35

pertaining to the safety of rail travel and


train operation. Statement 3 is correct: The BoP of an
The responsibility for safety in the working economy is calculated on the
and operation of the Railway rests solely principles of accountancy (double-
with the Railway Board and the Zonal entry bookkeeping) and looks like the
Railway authorities. The main task of the balance sheet of a company - every
Commission of Railway Safety is to direct, entry is shown either as credit (inflow)
advise and caution the Railway executives or debit (outflow). If there is a positive
with a view to ensure that all reasonable outcome at the end of the year, the
precautions are taken in money is automatically transferred to
regard to the soundness of rail construction the foreign exchange reserves of the
and safety of train operation economy. And if there is any negative
Statement 2 is correct: The outcome, the same foreign exchange is
commissioner is charged with certain drawn from the country’s forex
statutory functions and duties as laid reserves.
down in the Railway Act (1989), which Statement 4 is correct: If the forex
are of an inspectorial, investigatory & reserves are not capable of fulfilling
advisory nature. the negativity created by the BoP, it
• to inspect new railways with a view is known as a BoP crisis, and the
to determine whether they are fit to economy tries different means to solve
be opened for the public carriage of the crisis in which going for forex help
passengers and to report thereon to from the IMF is the last resort.
the Central Government as
required by or under this Act;
• to make such periodical or other
inspections of any Railway or of
any rolling stock used thereon as
the Central Government may
direct;
• to make an inquiry under this Act
into the cause of any accident on a
Railway;
• to perform such other duties as are
imposed on him by this Act or any
other enactment for the time being
in force relating to Railways

100. Ans: C
Exp:
Statement 1 is correct: The Balance
of Payment (BoP) indeed represents
the overall transactions an economy
engages in with the external world,
encompassing both current and
capital accounts. It might be
favourable or unfavourable for the
economy.
Statement 2 is incorrect: Negativity
of the BoP does not mean it is
unfavourable. A negative BoP is
unfavourable for an economy if the
economy lacks the means to fill the gap
of negativity.

https://upscpdf.com/ https://upscpdf.com/
https://t.me/visioniastestseries2024

You might also like